Puolimutkateisti on luvannut kääntyä kreationistiksi, jos joku todistaa evoluution mahdottomaksi.

EVOLUUTIOTEORIA ON NYKYÄÄN HELPPO KUMOTA

Evoluutioteorian voi kumota nykyään helposti vain yhdellä lauseella:

Jokainen syntyvä lapsi tuo 100-200 geenivirhettä perimään ja koska roska-DNA:ta ei ole, johtavat nämä geenivirheet väistämättömään ja nopeaan geneettiseen rappeutumiseen.

Ja koska evoluutiota ei siis täten tapahdu, eikä ole koskaan tapahtunutkaan, on luominen ainoa looginen johtopäätelmä. Raamattu on totta ensimmäisestä viimeiseen jakeeseen. Se on Jumalan Sanaa, eikä sen arvovaltaa saa väheksyä.

Johtavat evoluutiobiologit:

"If much more than 3% of genome is functional, evolution becomes primarily a destructive process." (PZ Myers, 2015)

"If any more than a small percentage of DNA has a meaningful sequence, such a mutation rate causes more than 1-2 harmful mutations per generation, and evolution becomes primarily a destructive process." (Larry Moran, 2014)

"Mutational load considerations lead to the conclusion that the functional fraction within the human genome cannot exceed 25%, and is probably considerably lower." (Dan Graur, 2017)

Nykytiede on osoittanut, ettei mitään roska-DNA:ta ole, joten evoluutio on tuhoava eli rappeuttava prosessi:

https://www.researchgate.net/publication/269187480_Non-coding_RNAs_Biological_functions_and_applications?fbclid=IwAR2RYi1Zw99F_-87r8sj0Q3z1FqHJQ19dCBHZUwDbSx2jaV4GPsBLO4ThRY

"In addition, there has been an explosion of research addressing possible functional roles for the other 98% of the human genome that does not encode proteins. In fact, >90% of the human genome is likely to be transcribed yielding a complex network of overlapping transcripts that include tens of thousands of long RNAs with little or no protein forming capacity; they are collectively called non-coding RNA."

Seisotko Puolimutkateisti sanojesi takana? Evoluutioteoria on nyt kumottu.

134

<50

    Vastaukset

    Anonyymi (Kirjaudu / Rekisteröidy)
    5000
    • Odotetaan vielä muutama päivä, että myös yliopistot huomaavat evoluutioteorian kumoutuneen.

      Mikä muuten on sen tilalle tullut uusi teoria biodiversiteetin kehityksestä? Missä siihen voi tutustua? Mitkä ovat sen falsifiointikriteerit?

      • Pysypä asiassa. Aloitukseni tieteelliset faktat kumoavat evoluutioteorian ja Puolimutkateistin tulisi seistä sanojensa takana.


      • Anonyymi
        RaamattuOnTotuus kirjoitti:

        Pysypä asiassa. Aloitukseni tieteelliset faktat kumoavat evoluutioteorian ja Puolimutkateistin tulisi seistä sanojensa takana.

        Ja vaikka kaatuisi, ei luominen muutu todeksi.


      • RaamattuOnTotuus kirjoitti:

        Pysypä asiassa. Aloitukseni tieteelliset faktat kumoavat evoluutioteorian ja Puolimutkateistin tulisi seistä sanojensa takana.

        " Pysypä asiassa. Aloitukseni tieteelliset faktat kumoavat evoluutioteorian ja Puolimutkateistin tulisi seistä sanojensa takana."

        Tee paperi niistä tieteellisista faktoistasi ja lähetä se Natureen tai Scienceen.

        Jos aikuisten oikeasti uskot, että harhaiset väutteesi kaatavat evoluutioteorian, olet vielä pahemmin sekaisin kuin luulinkaan. Jos taas tarkoitit tuon trollaamiseksi, niin ei siitäkään psiteitä heru. Tiedät itsekin, että jos tarjoaisit väitettäsi ammattitutkijoille, sinut naurettaisiin pihalle.

        T: Sarvikuono.


      • Anonyymi
        RaamattuOnTotuus kirjoitti:

        Pysypä asiassa. Aloitukseni tieteelliset faktat kumoavat evoluutioteorian ja Puolimutkateistin tulisi seistä sanojensa takana.

        Pysypä totuudessa:

        "Aloitukseni tieteelliset faktat kumoavat evoluutioteorian.."



        TH


      • Anonyymi
        Anonyymi kirjoitti:

        Pysypä totuudessa:

        "Aloitukseni tieteelliset faktat kumoavat evoluutioteorian.."



        TH

        Viestin pääosa karkasi, sorry.

        Vain evoluutiobiologien konsensus voi ottaa pitävän kannan evoluutioteorian oikeellisuuteen. Heidän nykyinen kantansa on vahvistava eikä kumoava.


      • RaamattuOnTotuus kirjoitti:

        Pysypä asiassa. Aloitukseni tieteelliset faktat kumoavat evoluutioteorian ja Puolimutkateistin tulisi seistä sanojensa takana.

        Teoriaa ei ymmärtääkseni voi kumota ilman korvaavaa parempaa teoriaa, joten saapi suorittaa. ✍🏻


      • Anonyymi
        RaamattuOnTotuus kirjoitti:

        Pysypä asiassa. Aloitukseni tieteelliset faktat kumoavat evoluutioteorian ja Puolimutkateistin tulisi seistä sanojensa takana.

        Eihän nuo mitään kumoa. Oletko edes opiskellut tiedettä?


      • RR: "Odotetaan vielä muutama päivä, että myös yliopistot huomaavat evoluutioteorian kumoutuneen."

        On se kyllä sitten hienoa kun Suomi mainitaan maailmalla. Suomalainen pseudotieteiljä kaatoi evoluutioteorian yhdellä lausella ...


      • Anonyymi

        Pseudotieteilijä kaatoi evoluutioteorian ,ja vieläpä tuomalla todisteeksi irrallisia lauseita tutkimuksista, jotka eivät kumoa evoluutiota.


      • Anonyymi

        Toisaalta pseudotieteilijä myöntää nyt evoluution olemassaolon, vaikkei aivan vielä ymmärräkään sen kokonaisuutta.

        "Nykytiede on osoittanut, ettei mitään roska-DNA:ta ole, joten evoluutio on tuhoava eli rappeuttava prosessi"

        - Knark5


      • Anonyymi
        puolimutkateisti kirjoitti:

        RR: "Odotetaan vielä muutama päivä, että myös yliopistot huomaavat evoluutioteorian kumoutuneen."

        On se kyllä sitten hienoa kun Suomi mainitaan maailmalla. Suomalainen pseudotieteiljä kaatoi evoluutioteorian yhdellä lausella ...

        Evoluutioteoria on teoria. Teorian nimi sadulle niemeltä evoluutio. Evoluutiosatu ei edes ala. Sillä ei ole alkua. Satu muuttuu eri aikakausina erilaiseksi. Darwin oli ensimmäisiä satuilijoita, jonka satukirjaa ei moni ole lukenut, mutta johon moni hyvin viisas, arvonsa tunteva viittaa. Onhan Darwinin serkkujen erilaiset nokat merkki pysyvästä lajiutumisesta. Hurjasta informaation kasvusta. Samoin kuin erilaiset koirarodut, jotka on jalosteta pysyvästi sairaiksi. Satu jatku ja muuta muotoaan. Pysy hereillä, ettet jää tarinan käänteistä paitsi. Kohta laboratoriossa luodaan uuta elämää ja ufoihin saadaan yhteys.


      • Anonyymi kirjoitti:

        Evoluutioteoria on teoria. Teorian nimi sadulle niemeltä evoluutio. Evoluutiosatu ei edes ala. Sillä ei ole alkua. Satu muuttuu eri aikakausina erilaiseksi. Darwin oli ensimmäisiä satuilijoita, jonka satukirjaa ei moni ole lukenut, mutta johon moni hyvin viisas, arvonsa tunteva viittaa. Onhan Darwinin serkkujen erilaiset nokat merkki pysyvästä lajiutumisesta. Hurjasta informaation kasvusta. Samoin kuin erilaiset koirarodut, jotka on jalosteta pysyvästi sairaiksi. Satu jatku ja muuta muotoaan. Pysy hereillä, ettet jää tarinan käänteistä paitsi. Kohta laboratoriossa luodaan uuta elämää ja ufoihin saadaan yhteys.

        Onnittelen! Todistit juuri 6000 vuotta sitten tapahtuneen Luomisen ja kumosit valheet miljoonista vuosista. Nyt voit mennä nukkumaan, jos tämän pommin jälkeen enää maltat.


      • RaamattuOnTotuus kirjoitti:

        Pysypä asiassa. Aloitukseni tieteelliset faktat kumoavat evoluutioteorian ja Puolimutkateistin tulisi seistä sanojensa takana.

        Niin mikä se vaihtoehto sitten on? Helluntaihurmosko? Etkö itse näe missiosi irvokkuutta?


      • RaamattuOnTotuus kirjoitti:

        Pysypä asiassa. Aloitukseni tieteelliset faktat kumoavat evoluutioteorian ja Puolimutkateistin tulisi seistä sanojensa takana.

        Anteeksi nyt että takerrun pikkuseikkoihin, mutta en ole nähnyt yhdessäkään aloituksessasi minkäänlaista häivähdystäkään sellaisesta tieteellisestä faktasta, joka kumoaisi evoluutioteorian.

        En ole myöskään nähnyt sinusta häivähdystäkään muualla kuin täällä ja blogissasi. Vertauskuvallisesti sanoen olet suurella vaivalla saanut pystyyn yhden ainoan seinän ja luulet rakentaneesi palatsin.


      • poikjuhg kirjoitti:

        Anteeksi nyt että takerrun pikkuseikkoihin, mutta en ole nähnyt yhdessäkään aloituksessasi minkäänlaista häivähdystäkään sellaisesta tieteellisestä faktasta, joka kumoaisi evoluutioteorian.

        En ole myöskään nähnyt sinusta häivähdystäkään muualla kuin täällä ja blogissasi. Vertauskuvallisesti sanoen olet suurella vaivalla saanut pystyyn yhden ainoan seinän ja luulet rakentaneesi palatsin.

        >Vertauskuvallisesti sanoen olet suurella vaivalla saanut pystyyn yhden ainoan seinän ja luulet rakentaneesi palatsin.

        Yhden seinän kakkimajaan, kuten Kari Heiskanen lausui Velipuolikuun asuntolainamainoksessa afrikkalaisille.


      • Anonyymi
        RepeRuutikallo kirjoitti:

        Onnittelen! Todistit juuri 6000 vuotta sitten tapahtuneen Luomisen ja kumosit valheet miljoonista vuosista. Nyt voit mennä nukkumaan, jos tämän pommin jälkeen enää maltat.

        Maailmankaikkeus ei laajenen. Laajeneminen on illuusio. Tähdet tippuvat kohta taivaalta.

        https://www.space.com/13094-accelerating-universe-dark-energy-illusion.html

        Lehti on tieteelisesti vertaisarvioitu lehti.


      • Anonyymi
        poikjuhg kirjoitti:

        Anteeksi nyt että takerrun pikkuseikkoihin, mutta en ole nähnyt yhdessäkään aloituksessasi minkäänlaista häivähdystäkään sellaisesta tieteellisestä faktasta, joka kumoaisi evoluutioteorian.

        En ole myöskään nähnyt sinusta häivähdystäkään muualla kuin täällä ja blogissasi. Vertauskuvallisesti sanoen olet suurella vaivalla saanut pystyyn yhden ainoan seinän ja luulet rakentaneesi palatsin.

        Revi tuosta.
        Usean osan systeemit
        http://www.intelligentdesign.fi/sivut/suppeat-artikkelit/redusoimaton-monimutkaisuus-ja-darwinismi/
        http://www.intelligentdesign.fi/sivut/laajat-artikkelit/empiirista-nayttoa-suunnittelusta/
        http://www.intelligentdesign.fi/sivut/laajat-artikkelit/biokemian-haaste-evoluutioteorialle/


    • Anonyymi

      Minä lupaan sinulle 10 000 euroa kun kumoat evoluution ja todistat luomisen tieteellisessä julkaisussa.

    • 3,5 mrd v. elämän historiaa kumoaa väitteesi.

      • Anonyymi

        Tarkoitat noin 6000 vuotta.


      • Anonyymi kirjoitti:

        Tarkoitat noin 6000 vuotta.

        Naurettava väite. Suomikin on ollut asuttu lähes 11 000 vuotta.


      • Anonyymi kirjoitti:

        Tarkoitat noin 6000 vuotta.

        Se kertaa 583 000.


    • "EVOLUUTIOTEORIA ON NYKYÄÄN HELPPO KUMOTA"
      "Jokainen syntyvä lapsi tuo 100-200 geenivirhettä perimään"

      Ja näin on ilmeisesti ollut aina. Nykyään meitä on n. 7 700 000 000 ja ihmiskunta voi paremmin kuin ikinä. Mitään merkkiä populaation rappeutumisesta ei ole.

      Jos jollain on ollut vielä epäilystä sinun tieteellisyydestäsi, niin ei pitäisi olla enää. Olen itse umpimaallikko, mutta tiedän aivan varmasti, ettei evoluutioteoria tuolla kumoudu. Tieteen suhteen olet pihalla kuin lumiukon pippeli.

    • Anonyymi

      Evoluutiosatu on keksitty satu eli ei voi olla totta. Raamattu on totta.

      • Anonyymi

        Ja raamatun totuus on ihmisten kokoaman kokoelmateoksen totuus. Tarinoita ja sepitteitä ajalta jolloin kameli oli nopein kulkuneuvo, ja aurinko heräsi aamulla ja meni nukkumaan illalla.


    • Anonyymi

      "EVOLUUTIOTEORIA ON NYKYÄÄN HELPPO KUMOTA"

      Ah kuinka kaukana ovat reaalimaailman rannat. Taikauskoiseen lahkoon kuuluva sekopää huutaa isoilla kirjaimilla, että luonnontieteellinen teoria on helppo kumota. Tietenkin näitä sekopäitä löytyy, etenkin nyt kun mielenterveyshoidon laitospaikkoja on vahvasti karsittu.

      Mutta kun tuo ( sanat eivät riitä tuntemuksiani kuvailemaan) toimii pienten lasten opettajana. Sille pitäisi oikeasti laittaa stoppi. Itse siirtäisin omat lapseni toiseen kouluun tai ottaisin pakon edessä ennemmin kotiopetukseen, kuin antaisin tuollaisen totaalisesti realiteettien tajunsa kadottaneen ihmisen opettaa heitä. Mieshän on täysin sekopää.

      • Niinpä. Mieleeni tulee avauksen tehneestä nimimerkistä lähinnä kylähullu huru-ukko, jonka todellisuuden mopo on karannut käsistä jo aikaa sitten ja kauaksi.


      • Anonyymi
        puolimutkateisti kirjoitti:

        Niinpä. Mieleeni tulee avauksen tehneestä nimimerkistä lähinnä kylähullu huru-ukko, jonka todellisuuden mopo on karannut käsistä jo aikaa sitten ja kauaksi.

        Kummasti huru-ukoilla kiinnostaa. Ole kenties vähän pervo. Myönnä pois, lisääntyminen kiinnostaa. Ehkä hieman sairaalloisesti. Evoluutiousko mutatoitumisineen saa henkisen orgasmin. Neula raapii kovalevyä ja järki seisoo. Syntyy uutta parempaa dataa. Tapahtuu itseohjautuvuus parempaan terveempää ja pitkäikäisempään tulevaisuuteen, jonka täyttää taustakohina.


      • Anonyymi
        Anonyymi kirjoitti:

        Kummasti huru-ukoilla kiinnostaa. Ole kenties vähän pervo. Myönnä pois, lisääntyminen kiinnostaa. Ehkä hieman sairaalloisesti. Evoluutiousko mutatoitumisineen saa henkisen orgasmin. Neula raapii kovalevyä ja järki seisoo. Syntyy uutta parempaa dataa. Tapahtuu itseohjautuvuus parempaan terveempää ja pitkäikäisempään tulevaisuuteen, jonka täyttää taustakohina.

        EI ole evoluutiolla tai sitä kuvaavalla teorialla mitään hätää, jos kretujen tietotaso on tätä :D


    • Anonyymi

      Evoluutioteoria on vain eläkettä odottavien velttojen opettajien professorien ja rappeutuneiden tiedemiesten keino pysyä virassaan .Jo tuhannet järkevät tiedemiehet alkavat tulla kaapeistaan, myöntäen,että tieteessä on liian paljon selittämättömiä asioita. Kun yksi asia selviää ,tuo se tullessaan kymmenen uutta kysymystä. Eli he eivät voi selittää mitään täydellisesti.

      • Anonyymi

        "Jo tuhannet järkevät tiedemiehet alkavat tulla kaapeistaan..."

        Kerrohan edes muutama nimi niistä "tuhansista järkevistä tiedemiehistä", jotka väittävät jonkun luonnontieteen yleisesti hyväksytyn teorian olevan väärä.
        Yhden asian selvittäminen tuo (weinbergin mukaan) kaksi uutta kysymystä, mutta ne kysymykset koskevat aina vain suppeampaa osa-aluetta ja tiedon tarkentumista.

        En voi omaamallani tiedolla selittää täydellisen tarkasti, kuinka paljon nopeammin Bottas kykenisi ajamaan mersullaan Imolan ratakierroksen kuin naapurin ukko pappatunturillaan. Minulla ei kuitenkaan ole mitään järjellistä epäilyä siitä, etteikö Bottas olisi nopeampi.


      • Anonyymi
        Anonyymi kirjoitti:

        "Jo tuhannet järkevät tiedemiehet alkavat tulla kaapeistaan..."

        Kerrohan edes muutama nimi niistä "tuhansista järkevistä tiedemiehistä", jotka väittävät jonkun luonnontieteen yleisesti hyväksytyn teorian olevan väärä.
        Yhden asian selvittäminen tuo (weinbergin mukaan) kaksi uutta kysymystä, mutta ne kysymykset koskevat aina vain suppeampaa osa-aluetta ja tiedon tarkentumista.

        En voi omaamallani tiedolla selittää täydellisen tarkasti, kuinka paljon nopeammin Bottas kykenisi ajamaan mersullaan Imolan ratakierroksen kuin naapurin ukko pappatunturillaan. Minulla ei kuitenkaan ole mitään järjellistä epäilyä siitä, etteikö Bottas olisi nopeampi.

        Jatkan.
        Lapsi hyvä, tieteessä on paljon selittämättömiä asioita. Jos ei olisi tutkijoilta loppuisivat työt. Se ei kuitenkaan tarkoita sitä, etteivätkö ne asiat, jotka on selitetty, pitäisi paikkaansa.
        Matematiikassa on taatusti enemmän sellaista, mistä minä en ymmärrä mitään kuin sellaista matematiikkaa, minkä minä hallitsen. Ei se silti tarkoita, että tietoni kertotaulusta tai potenssilaskusta olisivat vääriä.

        Valtaosa evoluutiotutkijoista on nuoria. Uusia opiskelijoita tulee alalle satoja vuodessa.


      • >Eli he eivät voi selittää mitään täydellisesti.

        Miksi pitäisi? Parempi osaselitys joka on ilmeisesti totta, kuin täydellinen selitys joka on selvästi väärä.


    • ROT älämölisee: "EVOLUUTIOTEORIA ON NYKYÄÄN HELPPO KUMOTA"

      Kas kummaa kun et silti ole kyennyt esittelemään ainoatakaan vertaisarvioitua tieteellistä julkaisua joka edes kyseenalaistaa evoluutioteorian johtopäätöksissään.

      ROT maanisessa suuruudenhulluudessaan: "Evoluutioteorian voi kumota nykyään helposti vain yhdellä lauseella:"

      ROT:in aivopieru: "Jokainen syntyvä lapsi tuo 100-200 geenivirhettä perimään ja koska roska-DNA:ta ei ole, johtavat nämä geenivirheet väistämättömään ja nopeaan geneettiseen rappeutumiseen."

      Se on tietääkseni totta että jokaisella syntyvällä lapsella on noin 100 mutaatiota genomissaan, joita ei ole kummallakaan vanhemmistaan.

      Rappeutuminen on puolestaan kreationistien uskonnollinen satu.

      ROT:in kreationista logiikkaa "Ja koska evoluutiota ei siis täten tapahdu, eikä ole koskaan tapahtunutkaan ..."

      Jospa nyt pseudotieteilijä esitteliset sen ensimmäisen vertaisarvioidun tieteellisen julkaisun joka edes kyseenalaistaa evoluutioteorian johtopäätöksissään.

      ROT: "... on luominen ainoa looginen johtopäätelmä. Raamattu on totta ensimmäisestä viimeiseen jakeeseen. Se on Jumalan Sanaa, eikä sen arvovaltaa saa väheksyä."

      Raamattu on vain ihmisten kirjoittama kokoelma myyttejä, jotka pohjautuvat vielä vanhempiin myytteihin. Raamatussa ei ole muita tosiasioita kuin muutama viite historiallisiin henkilöihin tai paikkoihin.

      ROT : "Nykytiede on osoittanut, ettei mitään roska-DNA:ta ole, joten evoluutio on tuhoava eli rappeuttava prosessi:

      ROT:in linkittämässä kirjallisuusarviossa (literature review) todetaan:

      "In addition, there has been an explosion of research addressing possible functional roles for the other 98% of the human genome that does not encode proteins. In fact, >90% of the human genome is likely to be transcribed yielding a complex network of overlapping transcripts that include tens of thousands of long RNAs with little or no protein forming capacity; they are collectively called non-coding RNA."

      Vaikka 90% genomista transkriptoitaisiinkin, niin se ei tarkoita että kaikilla transkriptioilla olisi funktio. Ja viimeisimpien tutkimusten mukaan suurin osa soluissa tuotetuista transkripteistä ovatkin ilman funtiota, eräänlaista "transkriptionaalista meteliä". Tässä yksi viimevuoden marraskuussa julkaistu tutkimus, joka toteaa johtopäätöksissään:

      "We assembled the sequences from deep RNA sequencing experiments by the Genotype-Tissue Expression (GTEx) project, to create a new catalog of human genes and transcripts, called CHESS. The new database contains 42,611 genes, of which 20,352 are potentially protein-coding and 22,259 are noncoding, and a total of 323,258 transcripts. These include 224 novel protein-coding genes and 116,156 novel transcripts. We detected over 30 million additional transcripts at more than 650,000 genomic loci, nearly all of which are likely nonfunctional, revealing a heretofore unappreciated amount of transcriptional noise in human cells."

      "The overall picture that emerges from this analysis is that the cell is a relatively inefficient machine, transcribing more DNA into RNA than it needs. Ever since the discovery of introns [54, 55], we have known that genomes contain large regions that appear to have no function. Based on the results described here, it appears that nearly 99% of the transcriptional variety produced in human cells has no apparent function, although most of these variants appear at such low levels that they cumulatively account for only 32% of transcriptional activity."

      Eli 99% ihmisen soluissa tuotetuilla transkriptioilla ei näytä olevan mitään funktiota.

      https://genomebiology.biomedcentral.com/articles/10.1186/s13059-018-1590-2

      ROT: "Seisotko Puolimutkateisti sanojesi takana?"

      Tottakai.

      ROT: "Evoluutioteoria on nyt kumottu."

      Se on kumottu vasta sitten kun tiedeyhteisö hyväksynyt selitysvoimaisemman tieteellisen teorian evoluutioteorian tilalle.

      Uskonnollis-pseudotieteellinen aivopierusi ei todista muuta kuin sen että olet tiedettä ymmärtämätön kreationisti.

      • Nykyään löytyy runsaasti tutkimustietoa siitä, että yli 90% ihmisen genomista luetaan transkriptioon. Tyypillisesti siitä koodataan erilaisia RNA-molekyylejä. Jokainen vähänkään molekyylibiologiaa tunteva tajuaa, ettei RNA-molekyylejä turhan takia rakenneta; niillä on useita tärkeitä säätely- ja viestinvälitystehtäviä sekä solussa että solujen välillä.

        https://link.springer.com/article/10.1007/s10620-017-4506-1
        https://phys.org/news/2017-02-mysterious-scientists-dark-genome.html


      • Anonyymi
        RaamattuOnTotuus kirjoitti:

        Nykyään löytyy runsaasti tutkimustietoa siitä, että yli 90% ihmisen genomista luetaan transkriptioon. Tyypillisesti siitä koodataan erilaisia RNA-molekyylejä. Jokainen vähänkään molekyylibiologiaa tunteva tajuaa, ettei RNA-molekyylejä turhan takia rakenneta; niillä on useita tärkeitä säätely- ja viestinvälitystehtäviä sekä solussa että solujen välillä.

        https://link.springer.com/article/10.1007/s10620-017-4506-1
        https://phys.org/news/2017-02-mysterious-scientists-dark-genome.html

        Luin molemmat linkittämäsi tutkimukset. Molemmissa kyllä esiintyi luku 90 %, mutta ei todellakaan väittämässäsi yhteydessä. Alla lainaus jälkimäisestä linkistä:
        ""For example, in genome-wide association studies for common diseases, such as diabetes, cancer and autism, 90 percent of the disease-associated DNA variants are in the noncoding DNA."

        Tulkitsetko sinä tuon tarkoittavan, että yli 90 % genomista luetaan transkriptioon? Minä en tulkitse, eikä kukaan muukaan englantia osaava täysipäinen ihminen. Toisen tutkimuksen maininta yhtä kaukana väitteestäsi.

        Kirjoittelet aamuyön tunteina valheita tutkijoiden tutkimuksista ja toivot, ettei kukaan viitsi lukea niitä. Vai oletko tosiaan niin tyhmä, ettet tajunnut, mitä artikkeleissa luki? Tätä olen joutunut ihmettelemään useita kertoja sinun kommentoituasi tieteellisiä artikkeleja.

        Noloa ROT, noloa.


      • RaamattuOnTotuus kirjoitti:

        Nykyään löytyy runsaasti tutkimustietoa siitä, että yli 90% ihmisen genomista luetaan transkriptioon. Tyypillisesti siitä koodataan erilaisia RNA-molekyylejä. Jokainen vähänkään molekyylibiologiaa tunteva tajuaa, ettei RNA-molekyylejä turhan takia rakenneta; niillä on useita tärkeitä säätely- ja viestinvälitystehtäviä sekä solussa että solujen välillä.

        https://link.springer.com/article/10.1007/s10620-017-4506-1
        https://phys.org/news/2017-02-mysterious-scientists-dark-genome.html

        ROT: "Nykyään löytyy runsaasti tutkimustietoa siitä, että yli 90% ihmisen genomista luetaan transkriptioon."

        Kyllä mutta vielä ei tiedetä mitkä transkriptioista ovat toiminnallisia. Asiaa tutkitaan ja aivan viimeaikaiset tutkimukset ovat osoittaneet että suuri osa niistä on ei-toiminnallisia.

        ROT: "Tyypillisesti siitä koodataan erilaisia RNA-molekyylejä. Jokainen vähänkään molekyylibiologiaa tunteva tajuaa, ettei RNA-molekyylejä turhan takia rakenneta; niillä on useita tärkeitä säätely- ja viestinvälitystehtäviä sekä solussa että solujen välillä."

        Ja sinähän et molekyylibiologiaa tunne kun jatkuvasti aivopiereskelet aiheesta.

        Todellisuudessa viimeaikaiset tutkimukset ovat osoittaneet että soluissa tapahtuu paljon turhia RNA-molekyylien transkriptioita ja muutenkin solujen keskeisimmät molekylaariset toiminnot eivät ole niin tarkkoja kuin on tähän saakka ajateltu. Tässä yksi tutkimus, joka on julkaistu tämän vuoden maaliskuussa. Tutkimus toteaa seuraavat johtopäätökset erityisesti vaihtoehtoisten transkription initiaatioiden (Alternative Transcriptional Initiation, ATI) kohdalta:

        "Alternative transcriptional initiation (ATI) refers to the frequent observation that one gene has multiple transcription start sites (TSSs). Although this phenomenon is thought to be adaptive, the specific advantage is rarely known. Here, we propose that each gene has one optimal TSS and that ATI arises primarily from imprecise transcriptional initiation that could be deleterious. This error hypothesis predicts that (i) the TSS diversity of a gene reduces with its expression level; (ii) the fractional use of the major TSS increases, but that of each minor TSS decreases, with the gene expression level; and (iii) cis-elements for major TSSs are selectively constrained, while those for minor TSSs are not. By contrast, the adaptive hypothesis does not make these predictions a priori. Our analysis of human and mouse transcriptomes confirms each of the three predictions. These and other findings strongly suggest that ATI predominantly results from molecular errors, requiring a major revision of our understanding of the precision and regulation of transcription."

        Ja tutkijoiden mukaan ATI-mekanismiin liittyvät epätarkkuudet ja virheet ovat linjassa myös muiden viime aikaisten havaintojen kanssa siitä että monet transkriptionaalista monimuotoisuutta lisäävät mekanimismit ovatkn suurelta osin molekyylivirheiden seurauksia eivätkä ole osa toiminnallisia adaptiivisia säätelymekanismeja.

        "Our results on ATI echo recent findings about a number of phenomena that increase transcriptome diversity, including alternative polyadenylation, alternative splicing, and several forms of RNA editing. They have all been shown to be largely the results of molecular errors instead of adaptive regulatory mechanisms. Together, these findings reveal the astonishing imprecision of key molecular processes in the cell, contrasting the common view of an exquisitely perfected cellular life."

        https://journals.plos.org/plosbiology/article?id=10.1371/journal.pbio.3000197

        Molemmat linkittämäsi artikkelit ovat katsauksia nykyisiin tutkimuksiin joissa pyritään selvitetään mitkä transkriptioista ovat toiminnallisia ja mitkä eivät. Kumpikaan niistä ei väitä että kaikki transkriptiot ovat toiminnallisia.


      • Anonyymi
        RaamattuOnTotuus kirjoitti:

        Nykyään löytyy runsaasti tutkimustietoa siitä, että yli 90% ihmisen genomista luetaan transkriptioon. Tyypillisesti siitä koodataan erilaisia RNA-molekyylejä. Jokainen vähänkään molekyylibiologiaa tunteva tajuaa, ettei RNA-molekyylejä turhan takia rakenneta; niillä on useita tärkeitä säätely- ja viestinvälitystehtäviä sekä solussa että solujen välillä.

        https://link.springer.com/article/10.1007/s10620-017-4506-1
        https://phys.org/news/2017-02-mysterious-scientists-dark-genome.html

        "Jokainen vähänkään molekyylibiologiaa tunteva tajuaa, ettei RNA-molekyylejä turhan takia rakenneta..."

        Olipahan taas aivopieru.


      • Anonyymi
        Anonyymi kirjoitti:

        Luin molemmat linkittämäsi tutkimukset. Molemmissa kyllä esiintyi luku 90 %, mutta ei todellakaan väittämässäsi yhteydessä. Alla lainaus jälkimäisestä linkistä:
        ""For example, in genome-wide association studies for common diseases, such as diabetes, cancer and autism, 90 percent of the disease-associated DNA variants are in the noncoding DNA."

        Tulkitsetko sinä tuon tarkoittavan, että yli 90 % genomista luetaan transkriptioon? Minä en tulkitse, eikä kukaan muukaan englantia osaava täysipäinen ihminen. Toisen tutkimuksen maininta yhtä kaukana väitteestäsi.

        Kirjoittelet aamuyön tunteina valheita tutkijoiden tutkimuksista ja toivot, ettei kukaan viitsi lukea niitä. Vai oletko tosiaan niin tyhmä, ettet tajunnut, mitä artikkeleissa luki? Tätä olen joutunut ihmettelemään useita kertoja sinun kommentoituasi tieteellisiä artikkeleja.

        Noloa ROT, noloa.

        Hieman tarkennusta
        https://www.researchgate.net/publication/269187480_Non-coding_RNAs_Biological_functions_and_applications?fbclid=IwAR2RYi1Zw99F_-87r8sj0Q3z1FqHJQ19dCBHZUwDbSx2jaV4GPsBLO4ThR


    • "Jokainen syntyvä lapsi tuo 100-200 geenivirhettä perimään ja koska roska-DNA:ta ei ole, johtavat nämä geenivirheet väistämättömään ja nopeaan geneettiseen rappeutumiseen."

      Yrität (ilmeisesti) saada sen kuulostamaan siltä, että jokaisessa sukupolvessa tulee yli sata mutaatiota lisää. Väärin, se on joka sukupolvessa tuo määrä. Kaikki mutaatiot eivät myöskään ole haitallisia.
      Katso ympärillesi - näyttääkö siltä että ihmiskunta on rappeutumassa? Moraalisesti ehkä, mutta se ei liity genetiikkaan.

      • Anonyymi

        1.Ihmisen genomiin tulee joka sukupolvessa liian paljon lievästi haitallisia mutaatioita. 50-luvulla oletettiin, että vahingollisten mutaatioiden määrä olisi 0.12-0.30 mutaatiota yksilöä kohden sukupolvessa. Tämä oli välttämätön oletus populaatiogeneettisissä laskuissa jotta valinta voisi karsia haitallisia mutaatioita pois ja ihminen voisi kehittyä. Viimeaikaisten tutkimusten mukaan jo pelkästään nukleotidikorvautumisia tapahtuu 75-175 yksilöä kohden sukupolvessa ja tähän päälle Sanford listaa tuoretta tutkimusta muiden mutaatiotyyppien (satelliittimutaatiot, deleetiot, duplikaatiot/insertiot, inversiot/translokaatiot, konversiot, mitokondrion mutaatiot) yhteensä vielä suuremmasta määrästä. Sanford siis esittää pitkälti vastaansanomatonta empiiristä aineistoa mutaatioiden liian suuresta määrästä.
        2.Hyödylliset mutaatiot ovat hyvin harvinaisia. R. Fisher oletti luonnonvalintaa koskevissa laskuissaan (Fisherin fundamentaalinen teoreema), että hyödyllisiä ja haitallisia mutaatioita on saman verran. Modernin tutkimuksen mukaan kuitenkin vain joka miljoonas mutaatio on hyödyllinen. Sanford käyttää laskuissaan ja simulaatioissaan arviota, että joka tuhannes mutaatio olisi hyödyllinen. Eikä tässä vielä kaikki. Suurin osa haitallisista mutaatioista vaikuttaa eliön selviytymiseen vain tuhannesosien verran muihin selviytymistekijöihin verrattuna, joten geenien periytymistä hallitsee pääasiassa neutraali ajautuminen. Suurin osa haitallisista mutaatioista on siis käytännössä luonnonvalinnan ulottumattomissa.
        3.Luonnonvalinta on tehoton mekanismi valitsemaan haitallisia mutaatioita pois lajin genomista. Tämä johtuu siitä, että luonnonvalinta ei valitse yksittäisiä nukleotidipaikkoja tai edes geenejä vaan tietenkin yksilöitä, joissa ilmentyy kerralla koko niiden geneettinen ja epigeneettinen informaatio. Luonnonvalinta on siis tehoton poistamaan yksittäisiä kirjoitusvirheitä koska sen on valittava kokonaisia kirjoja. Lisäksi tiedetään, että siinä missä yksinkertaisilla ominaisuuksilla (kuten veriryhmä) on usein jopa 100% periytyvyys, monimutkaiset ominaisuudet kuten kelpoisuus ovat vain heikosti periytyviä (jopa 0,4%). Tämä johtuu siitä, että kokonaiskelpoisuus sulkee sisäänsä suuren määrän muita tekijöitä, jotka ovat peräisin sekä yksilön että sen ympäristön erilaisista piirteistä. Huonojen yksilöiden poistaminen vaikuttaa siis hyvin vähän huonojen genomien poistamiseen. Lisäksi lisääntyminen on luonnossa hyvin satunnainen prosessi, eivätkä kelpoisemmat yksilöt tyypillisesti lisäänny merkittävästi suuremmalla todennäköisyydellä. Tämän lisäksi kohinaa aiheuttavat sukusolujen otannan satunnaiset heilahtelut.

        Näiden tekijöiden yhteisvaikutuksena voidaan todeta, että ihmislajin ja useimpien muiden lajien kelpoisuus laskee, ne eivät kehity. Ongelma on populaatiogenetiikan alalla tunnustettu mutta siihen ei haluta uskoa. Sanfordin argumentti kohtaa hiljaisuutta. Johtopäätös kuitenkin on, että evoluutiomekanismi rappeuttaa ihmisen genomia, ihminen ei siis kehity paremmaksi, eikä ole kehittynyt, vaan rappeutuu ja on rappeutunut.

        ID tutkimuksen kannalta Sanfordin tulokset sopivat hyvin sekä M. Behen argumenttiin evoluutiomekanismin rajasta, D. Axen ja A. Gaugerin kokeisiin ja laskuihin proteiinien evoluution mahdottomuudesta, että J. Spetnerin tuoreessa kirjassaan lanseeraamaan ei-satunnaiseen evoluutioon, jossa Spetner pitää todennäköisenä, että tietyissä tilanteissa havaittu lajien nopea sopeutuminen on ennalta ohjelmoitua genomiin ja/tai epigenomiin.


      • Anonyymi
        Anonyymi kirjoitti:

        1.Ihmisen genomiin tulee joka sukupolvessa liian paljon lievästi haitallisia mutaatioita. 50-luvulla oletettiin, että vahingollisten mutaatioiden määrä olisi 0.12-0.30 mutaatiota yksilöä kohden sukupolvessa. Tämä oli välttämätön oletus populaatiogeneettisissä laskuissa jotta valinta voisi karsia haitallisia mutaatioita pois ja ihminen voisi kehittyä. Viimeaikaisten tutkimusten mukaan jo pelkästään nukleotidikorvautumisia tapahtuu 75-175 yksilöä kohden sukupolvessa ja tähän päälle Sanford listaa tuoretta tutkimusta muiden mutaatiotyyppien (satelliittimutaatiot, deleetiot, duplikaatiot/insertiot, inversiot/translokaatiot, konversiot, mitokondrion mutaatiot) yhteensä vielä suuremmasta määrästä. Sanford siis esittää pitkälti vastaansanomatonta empiiristä aineistoa mutaatioiden liian suuresta määrästä.
        2.Hyödylliset mutaatiot ovat hyvin harvinaisia. R. Fisher oletti luonnonvalintaa koskevissa laskuissaan (Fisherin fundamentaalinen teoreema), että hyödyllisiä ja haitallisia mutaatioita on saman verran. Modernin tutkimuksen mukaan kuitenkin vain joka miljoonas mutaatio on hyödyllinen. Sanford käyttää laskuissaan ja simulaatioissaan arviota, että joka tuhannes mutaatio olisi hyödyllinen. Eikä tässä vielä kaikki. Suurin osa haitallisista mutaatioista vaikuttaa eliön selviytymiseen vain tuhannesosien verran muihin selviytymistekijöihin verrattuna, joten geenien periytymistä hallitsee pääasiassa neutraali ajautuminen. Suurin osa haitallisista mutaatioista on siis käytännössä luonnonvalinnan ulottumattomissa.
        3.Luonnonvalinta on tehoton mekanismi valitsemaan haitallisia mutaatioita pois lajin genomista. Tämä johtuu siitä, että luonnonvalinta ei valitse yksittäisiä nukleotidipaikkoja tai edes geenejä vaan tietenkin yksilöitä, joissa ilmentyy kerralla koko niiden geneettinen ja epigeneettinen informaatio. Luonnonvalinta on siis tehoton poistamaan yksittäisiä kirjoitusvirheitä koska sen on valittava kokonaisia kirjoja. Lisäksi tiedetään, että siinä missä yksinkertaisilla ominaisuuksilla (kuten veriryhmä) on usein jopa 100% periytyvyys, monimutkaiset ominaisuudet kuten kelpoisuus ovat vain heikosti periytyviä (jopa 0,4%). Tämä johtuu siitä, että kokonaiskelpoisuus sulkee sisäänsä suuren määrän muita tekijöitä, jotka ovat peräisin sekä yksilön että sen ympäristön erilaisista piirteistä. Huonojen yksilöiden poistaminen vaikuttaa siis hyvin vähän huonojen genomien poistamiseen. Lisäksi lisääntyminen on luonnossa hyvin satunnainen prosessi, eivätkä kelpoisemmat yksilöt tyypillisesti lisäänny merkittävästi suuremmalla todennäköisyydellä. Tämän lisäksi kohinaa aiheuttavat sukusolujen otannan satunnaiset heilahtelut.

        Näiden tekijöiden yhteisvaikutuksena voidaan todeta, että ihmislajin ja useimpien muiden lajien kelpoisuus laskee, ne eivät kehity. Ongelma on populaatiogenetiikan alalla tunnustettu mutta siihen ei haluta uskoa. Sanfordin argumentti kohtaa hiljaisuutta. Johtopäätös kuitenkin on, että evoluutiomekanismi rappeuttaa ihmisen genomia, ihminen ei siis kehity paremmaksi, eikä ole kehittynyt, vaan rappeutuu ja on rappeutunut.

        ID tutkimuksen kannalta Sanfordin tulokset sopivat hyvin sekä M. Behen argumenttiin evoluutiomekanismin rajasta, D. Axen ja A. Gaugerin kokeisiin ja laskuihin proteiinien evoluution mahdottomuudesta, että J. Spetnerin tuoreessa kirjassaan lanseeraamaan ei-satunnaiseen evoluutioon, jossa Spetner pitää todennäköisenä, että tietyissä tilanteissa havaittu lajien nopea sopeutuminen on ennalta ohjelmoitua genomiin ja/tai epigenomiin.

        Tähän olisi tietysti voinut lisätä koko tekstin lähteen, eli "Älykkään suunnitelman idea"-sivuston, josta koko teksti on suoraan copy-pastettu: http://www.intelligentdesign.fi/category/kaikki/id-teoria/

        Sivuston otsikkorivillä on marginaalisesti mainittu "Intelligent Design - teorian suomalainen portaali". Tosin teoriahan koko ID-idean taustalta puuttuu kokonaan, eikä sitä voida edes hypoteesiksikaan kelpuuttaa erään merkitsevän tekijän puuttumisen vuoksi.


      • Anonyymi
        Anonyymi kirjoitti:

        Tähän olisi tietysti voinut lisätä koko tekstin lähteen, eli "Älykkään suunnitelman idea"-sivuston, josta koko teksti on suoraan copy-pastettu: http://www.intelligentdesign.fi/category/kaikki/id-teoria/

        Sivuston otsikkorivillä on marginaalisesti mainittu "Intelligent Design - teorian suomalainen portaali". Tosin teoriahan koko ID-idean taustalta puuttuu kokonaan, eikä sitä voida edes hypoteesiksikaan kelpuuttaa erään merkitsevän tekijän puuttumisen vuoksi.

        Intelligent Design=the theory that life, or the universe, cannot have arisen by chance and was designed and created by some intelligent entity. Elämä tai maailmankaikkeus ei voi olla syntynyt sattumalta, ja sen on suunnitellut ja luonut jokin älykäs kokonaisuus.


      • Anonyymi
        Anonyymi kirjoitti:

        Intelligent Design=the theory that life, or the universe, cannot have arisen by chance and was designed and created by some intelligent entity. Elämä tai maailmankaikkeus ei voi olla syntynyt sattumalta, ja sen on suunnitellut ja luonut jokin älykäs kokonaisuus.

        "Intelligent Design=the theory that life, or the universe, cannot have arisen by chance and was designed and created by some intelligent entity."
        Vaikka monet englanninkielen sanakirjat käyttävätkin tässä yhteydessä sanaa "theory", niin ID ei ole tieteellistä teoriaa nähnytkään, sillä edelleenkään ei ole minkäänlaista havaintoa tästä "jostain älykkäästä kokonaisuudesta luojana ja suunnittelijana". Tästä samasta syystä ID ei pysty olemaan edes hypoteesi, koska sen kulmakivenä oleva yliluonnollinen entiteetti ei perustu mihinkään olemassa olevaan näyttöön, toisin kuin esimerkiksi laattatektoniikka, kvanttimekaniikka, evoluutioteoria ja yleinen suhteellisuusteoria. Tästä voit halutessasi väitellä, mutta valitettavasti tämä ei ole tieteen piirissä mikään mielipideasia.

        "Elämä tai maailmankaikkeus ei voi olla syntynyt sattumalta, ja sen on suunnitellut ja luonut jokin älykäs kokonaisuus."
        Väite jota et voi todistaa. Ja edelleenkin puuttuu ne havainnot tästä "älykkäästä suunnittelijasta ja luojasta".


      • Anonyymi
        Anonyymi kirjoitti:

        "Intelligent Design=the theory that life, or the universe, cannot have arisen by chance and was designed and created by some intelligent entity."
        Vaikka monet englanninkielen sanakirjat käyttävätkin tässä yhteydessä sanaa "theory", niin ID ei ole tieteellistä teoriaa nähnytkään, sillä edelleenkään ei ole minkäänlaista havaintoa tästä "jostain älykkäästä kokonaisuudesta luojana ja suunnittelijana". Tästä samasta syystä ID ei pysty olemaan edes hypoteesi, koska sen kulmakivenä oleva yliluonnollinen entiteetti ei perustu mihinkään olemassa olevaan näyttöön, toisin kuin esimerkiksi laattatektoniikka, kvanttimekaniikka, evoluutioteoria ja yleinen suhteellisuusteoria. Tästä voit halutessasi väitellä, mutta valitettavasti tämä ei ole tieteen piirissä mikään mielipideasia.

        "Elämä tai maailmankaikkeus ei voi olla syntynyt sattumalta, ja sen on suunnitellut ja luonut jokin älykäs kokonaisuus."
        Väite jota et voi todistaa. Ja edelleenkin puuttuu ne havainnot tästä "älykkäästä suunnittelijasta ja luojasta".

        Tyhjästä ei synny energiaaa eikä ainetta. Puhdas energia ei reagoi energiaan. Pimeää ainetta ei ole eikä energiaa. myös antimateria ja materia on tasapainossa. kahdeksan numeron tarkkuudella. Maailman syntyminen on termodynamiikan ja suhteellisuusteorian vastainen.

        Evoluutio on mahdottomuus.

        https://creation.com/laws-of-information-1
        https://creation.com/laws-of-information-2

        Solun sisältämä tieto ei synny tyhjästä.


      • Anonyymi
        Anonyymi kirjoitti:

        Tyhjästä ei synny energiaaa eikä ainetta. Puhdas energia ei reagoi energiaan. Pimeää ainetta ei ole eikä energiaa. myös antimateria ja materia on tasapainossa. kahdeksan numeron tarkkuudella. Maailman syntyminen on termodynamiikan ja suhteellisuusteorian vastainen.

        Evoluutio on mahdottomuus.

        https://creation.com/laws-of-information-1
        https://creation.com/laws-of-information-2

        Solun sisältämä tieto ei synny tyhjästä.

        "Tyhjästä ei synny energiaaa eikä ainetta."
        Voitko käydä kertomassa tämän saman fyysikoille, jotka tutkivat mm. kvanttifluktuaatiota, että he ovat täysin väärässä tutkimustensa kanssa.

        "Puhdas energia ei reagoi energiaan. Pimeää ainetta ei ole eikä energiaa. myös antimateria ja materia on tasapainossa. kahdeksan numeron tarkkuudella. Maailman syntyminen on termodynamiikan ja suhteellisuusteorian vastainen."
        Kuka nämä on todistanut sekä laskenut ja missä? Eli tieteelliset tutkimukset pöytään. Creation.com-sivusto ei ole tieteellinen lähde, varsinkin kun noissa kahdessa linkittämässäsi linkissä viitataan vain raamattuun sekä Journal of Creation-julkaisuun, joka puolestaan ei ole vertaisarvioitu tieteellinen julkaisu. Jos menet väitteinesi ja näiden lähteiden kanssa mihin tahansa yliopistoon fysiikan laitokselle, niin sinut nauretaan väitteinesi sieltä pihalle aika nopeasti.

        "Evoluutio on mahdottomuus."
        Ei liity millään tavalla maailman syntymiseen. Pystytkö selittämään jollain muulla tavalla esim. antibioottiresistenssin? Ja kuten jo aikaisemmin totesin, niin saat kyllä väitellä tästä niin paljon kuin haluat, mutta kyseessä ei ole mielipideasia, vaan luonnossa havaittu tosiasia. Lisäksi sinulta edelleen puuttuu näyttö yliluonnollisesta luojaentiteetistä. Vaikka evoluutioteoria tänään kumottaisiin, niin tuota tieteen aukkoa ei tulla paikkaamaan mystiikalla ja ihmeillä mistä ei edelleenkään ole havaintoja.

        "https://creation.com/laws-of-information-1
        https://creation.com/laws-of-information-2"
        Eivät toimi pätevinä lähteinä, koska tunnetusti näiltä sivustoilta löytyy niin hurja määrä lainauslouhintaa ja suoranaista valehtelua, ettei näitä lähteitä kelputettaisi ikinä tieteellisiin tutkimuksiinkaan.

        "Solun sisältämä tieto ei synny tyhjästä."
        Kuka näin on koskaan väittänytkään? Se vaatii tietyt elämän peruskomponentit (hiili ja vety), joita luonnossa esiintyy täysin riittävästi. Laboratoriokokeissakin on jo ajat sitten pystytty eri molekyylien keskinäisten reaktioiden kautta muodostamaan elämälle välttämättömiä aminohappoja, joten miksei se voisi tapahtua luonnollisestikin. Kyse on puhtaasti kemiallisista reaktioista.

        Sinulla on edelleen todistamatta se yliluonnollinen luojaentiteetti.


      • Anonyymi
        Anonyymi kirjoitti:

        "Tyhjästä ei synny energiaaa eikä ainetta."
        Voitko käydä kertomassa tämän saman fyysikoille, jotka tutkivat mm. kvanttifluktuaatiota, että he ovat täysin väärässä tutkimustensa kanssa.

        "Puhdas energia ei reagoi energiaan. Pimeää ainetta ei ole eikä energiaa. myös antimateria ja materia on tasapainossa. kahdeksan numeron tarkkuudella. Maailman syntyminen on termodynamiikan ja suhteellisuusteorian vastainen."
        Kuka nämä on todistanut sekä laskenut ja missä? Eli tieteelliset tutkimukset pöytään. Creation.com-sivusto ei ole tieteellinen lähde, varsinkin kun noissa kahdessa linkittämässäsi linkissä viitataan vain raamattuun sekä Journal of Creation-julkaisuun, joka puolestaan ei ole vertaisarvioitu tieteellinen julkaisu. Jos menet väitteinesi ja näiden lähteiden kanssa mihin tahansa yliopistoon fysiikan laitokselle, niin sinut nauretaan väitteinesi sieltä pihalle aika nopeasti.

        "Evoluutio on mahdottomuus."
        Ei liity millään tavalla maailman syntymiseen. Pystytkö selittämään jollain muulla tavalla esim. antibioottiresistenssin? Ja kuten jo aikaisemmin totesin, niin saat kyllä väitellä tästä niin paljon kuin haluat, mutta kyseessä ei ole mielipideasia, vaan luonnossa havaittu tosiasia. Lisäksi sinulta edelleen puuttuu näyttö yliluonnollisesta luojaentiteetistä. Vaikka evoluutioteoria tänään kumottaisiin, niin tuota tieteen aukkoa ei tulla paikkaamaan mystiikalla ja ihmeillä mistä ei edelleenkään ole havaintoja.

        "https://creation.com/laws-of-information-1
        https://creation.com/laws-of-information-2"
        Eivät toimi pätevinä lähteinä, koska tunnetusti näiltä sivustoilta löytyy niin hurja määrä lainauslouhintaa ja suoranaista valehtelua, ettei näitä lähteitä kelputettaisi ikinä tieteellisiin tutkimuksiinkaan.

        "Solun sisältämä tieto ei synny tyhjästä."
        Kuka näin on koskaan väittänytkään? Se vaatii tietyt elämän peruskomponentit (hiili ja vety), joita luonnossa esiintyy täysin riittävästi. Laboratoriokokeissakin on jo ajat sitten pystytty eri molekyylien keskinäisten reaktioiden kautta muodostamaan elämälle välttämättömiä aminohappoja, joten miksei se voisi tapahtua luonnollisestikin. Kyse on puhtaasti kemiallisista reaktioista.

        Sinulla on edelleen todistamatta se yliluonnollinen luojaentiteetti.

        "Sinulla on edelleen todistamatta se yliluonnollinen luojaentiteetti."

        Joku on tainnut joskus yrittää jotakin sellaista, että koska luoja on yliluonnolline, häntä ei voi todistaa - ja juuri se todistaa että hän on luonut kaiken. Olikohan tämä Mark5-logiikkaa?


      • Anonyymi
        Anonyymi kirjoitti:

        "Tyhjästä ei synny energiaaa eikä ainetta."
        Voitko käydä kertomassa tämän saman fyysikoille, jotka tutkivat mm. kvanttifluktuaatiota, että he ovat täysin väärässä tutkimustensa kanssa.

        "Puhdas energia ei reagoi energiaan. Pimeää ainetta ei ole eikä energiaa. myös antimateria ja materia on tasapainossa. kahdeksan numeron tarkkuudella. Maailman syntyminen on termodynamiikan ja suhteellisuusteorian vastainen."
        Kuka nämä on todistanut sekä laskenut ja missä? Eli tieteelliset tutkimukset pöytään. Creation.com-sivusto ei ole tieteellinen lähde, varsinkin kun noissa kahdessa linkittämässäsi linkissä viitataan vain raamattuun sekä Journal of Creation-julkaisuun, joka puolestaan ei ole vertaisarvioitu tieteellinen julkaisu. Jos menet väitteinesi ja näiden lähteiden kanssa mihin tahansa yliopistoon fysiikan laitokselle, niin sinut nauretaan väitteinesi sieltä pihalle aika nopeasti.

        "Evoluutio on mahdottomuus."
        Ei liity millään tavalla maailman syntymiseen. Pystytkö selittämään jollain muulla tavalla esim. antibioottiresistenssin? Ja kuten jo aikaisemmin totesin, niin saat kyllä väitellä tästä niin paljon kuin haluat, mutta kyseessä ei ole mielipideasia, vaan luonnossa havaittu tosiasia. Lisäksi sinulta edelleen puuttuu näyttö yliluonnollisesta luojaentiteetistä. Vaikka evoluutioteoria tänään kumottaisiin, niin tuota tieteen aukkoa ei tulla paikkaamaan mystiikalla ja ihmeillä mistä ei edelleenkään ole havaintoja.

        "https://creation.com/laws-of-information-1
        https://creation.com/laws-of-information-2"
        Eivät toimi pätevinä lähteinä, koska tunnetusti näiltä sivustoilta löytyy niin hurja määrä lainauslouhintaa ja suoranaista valehtelua, ettei näitä lähteitä kelputettaisi ikinä tieteellisiin tutkimuksiinkaan.

        "Solun sisältämä tieto ei synny tyhjästä."
        Kuka näin on koskaan väittänytkään? Se vaatii tietyt elämän peruskomponentit (hiili ja vety), joita luonnossa esiintyy täysin riittävästi. Laboratoriokokeissakin on jo ajat sitten pystytty eri molekyylien keskinäisten reaktioiden kautta muodostamaan elämälle välttämättömiä aminohappoja, joten miksei se voisi tapahtua luonnollisestikin. Kyse on puhtaasti kemiallisista reaktioista.

        Sinulla on edelleen todistamatta se yliluonnollinen luojaentiteetti.

        Proteiinien evoluution mahdottomuus

        Douglas Axe on tehnyt kokeellisen tutkimuksen, jonka mukaan jo 150 aminohapon ketjuista vain yksi 1062:sta voi laskostua vakaaseen biologisesti toimintavalmiiseen muotoon. Tietyn toiminnon saavuttamisen satunnaistodennäköisyys on noin 10-77. Maapallon resursseilla (1050 atomia, enintään 1018 sekuntia, toisaalta enintäänkin vain 1040 solua maapallon historian aikana) ei siis saada aikaan edes yhtä toiminnallista proteiinia puhtaasti sattuman tuloksena. Useimmat solun toiminnot vaativat vähintään kuusi toimivaa proteiinia, joten satunnaistodennäköisyydet ovat tarkastelun tässä vaiheessa jo tähtitieteellisen pieniä. Evoluutioteoria edellyttää lähtökohdakseen satunnaistarkoituksettomasti syntyneen alkusolun. Jo sellaisen vaatimien riittävän pitkien aminohappoketjujenkin satunnaissynty on kuitenkin mahdotonta, ja tämä proteiinitoiminnallisuustarkastelu osoittaa asiaan liittyvän myös valtavia lisäongelmia.

        Periaatteessa mahdollisten aminohappoketjujen joukkoa "karttakuvana" tarkasteltaessa osoittautuu siis, että toiminnallisten proteiinien "saarekkeiden" välissä on valtava toimimattomuuden "meri". Periaatteessa olisi silti ajateltavissa, että kahden toimivan saarekkeen välillä olisi pitkä pienin yksittäismuutoksin toisistaan saatavien toimivien proteiinien kehityspolku, joka lisäksi antaisi valintaedun koko mittansa yli. Onko sitten uskottavaa, että kahden näin kaukaisen saaren välillä todella olisi haitattomien välivaiheiden polku? Nimenomaan luonnontieteille perustavan tärkeä empiirinen näyttö osoittaa, että evoluutioteorian vaatimaa polkua "toimimattomuuden meren" yli ei edes helpoissa tapauksissa ole.


        Universumin resurssien rajallisuus

        Naturalistinen selittäminen siis kohtaa monia ongelmia. Nämä eivät suinkaan ole pelkkään asiantuntemattomuuteen perustuvia "pannukakkumaaperusteluja", vaan asioihin tarkemmin perehdyttäessä ongelmat entisestään terävöityvät. Jos evolutionististen skenaarioiden toteutumistodennäköisyyksien ylärajoja lainkaan arvioidaan, ne osoittautuvat häviävän pieniksi; kyse ei ole siitä, että pelkkä nykytilaan johtava reitti olisi epätodennäköinen, vaan, kuten biologisten keskinäisriippuvuuksien tarkasteleminen osoittaa, epätodennäköisyys koskee minkäänlaisen biosfäärin ohjaamatonta synty- ja kehitysmahdollisuutta. Tämä tarkoittaa sitä, että vuosimiljardienkaan tarjoamat resurssit eivät riitä vetämään evolutionismia epäuskottavuuden allikosta, ja ylettömän pitkillä aikajaksoilla brassailu onkin viime kädessä harhaista, tavallaan esitieteellistä uskomuksellisuutta, joka ei kestä siirtymää kvalitatiivisesta kvantitatiiviseen tarkasteluun.

        Matematiikassa tunnetaan paljon suurempiakin lukuja kuin esim. maailmankaikkeuden oletetusta n. 15 miljardin vuoden takaisesta alusta nykyhetkeen kuluneiden sekuntien määrän (suuruusluokkaa 1018) ja maailmankaikkeuden nykyarvion mukaan sisältämien alkeishiukkasten lukumäärän (enintään suuruusluokkaa 1087) tulo (siis enintään suuruusluokkaa 10105); samoin tunnetaan itseisarvoltaan paljon pienempiäkin lukuja kuin edellisen käänteisluku (10-105). Äärellisen maailman tarjoamat mahdollisuudet ovat välttämättä vain äärellisiä, kun taas kehitysskenaarioiden epäuskottavuudet voivat helpostikin luhistua "infinitesimaalisuuden mustaan aukkoon".

        Oletetun abiogeneesin ja evoluution käytettävissä oleviksi ajatelluilla resursseilla on itse asiassa matemaattisesti määrittyvät, selkeät ylärajat, ja kaikki havaittava olisi pystyttävä mahduttamaan niiden puitteisiin. Johtaviin ID-teoreetikkoihin lukeutuvan mm. matematiikan tohtorin William Dembskin tutkimusotteena onkin ollut havaittavien biologisten rakenteiden tarkoituksettoman syntytodennäköisyyden yläraja-arvioiden suhteuttaminen big bang -kosmologian mukaisten probabilististen resurssien määrään. Tämän lähestymistavan perustava logiikka on vertaisarvioitu matematiikan ja tilastotieteen alan väitöskirjana, joka on myös julkaistu kuulun brittiläisen Cambridgen yliopiston julkaisusarjassa (Cambridge Studies in Probability, Induction, and Decision Theory). Se on siis ajanmukaista eksaktia tiedettä.


    • Anonyymi

      "Ja koska evoluutiota ei siis täten tapahdu, eikä ole koskaan tapahtunutkaan, on luominen ainoa looginen johtopäätelmä. "

      Ja millä loogisella päätelmällä uskovaisten tarina muuttuu todeksi, jos leikitään että evoluutio yhtä-äkkiä lakkaisi olemasta?

    • Täytyy myöntää, että minäkin kumosin eilen vahingossa evoluutioteorian, vaikka en sitä tarkoittanut tehdä. Lainasin tässä kommentissa erästä eturivin kehitysbiologian tutkijaa, jonka mukaan käsitykset roska-DNA:sta ovat muuttuneet, eikä se ole 90% hyödytöntä tilkettä vaan siellä on säätelyalueita jotka ohjaavat viestijärjestelmän geenien kopiointia ja kertovat, missä ja kuinka paljon signaalimolekyylejä tuotetaan.

      https://keskustelu.suomi24.fi/t/16019193/dnan-virheensietokyky-ja-passiivinen-informaatio#comment-98924285

      • Kerro tuo PZ Myers:ille ja suututa hänet. Hänhän on todennut, että jos yli 3% genomista on toiminnallista, tulee evoluutiosta tuhoava prosessi.

        Hän on oikeassa.


      • Anonyymi
        RaamattuOnTotuus kirjoitti:

        Kerro tuo PZ Myers:ille ja suututa hänet. Hänhän on todennut, että jos yli 3% genomista on toiminnallista, tulee evoluutiosta tuhoava prosessi.

        Hän on oikeassa.

        Ei ole. Yksi tutkimus ei muuta konsensusta. Tietäisit sen itsekin aivan hyvion, jos ymmärtäisit tieteen teosta edes jotain. Mayersin lähtökohdat ja tulkinnat voidaan kiistää.
        Lisäksi on epävarmuutta siitä, kuinka suuri osa on toiminnallista ja mitä toiminnallisuudella missäkin yhteydessä tarkoitetaan, muuttaa tulkintaa.

        Suututa hänet? Tuskin oikea tutkija on yhtä lapsellinen kuin sinä ja saa itkupotkuraivareita, jos hänen tutkimustaan arvostellaan. Kritiikki kuuluu olennaisena osana tieteelliseen prosessiin. Vakavaksi tarkoitettuun tutkimuspaperiin oli vertaisarvioija kirjoittanut kommentin: "Ei edes väärin." Saman kommentin voisi laittaa useimpiin ulostuloihisi.


      • Anonyymi

        "Lisäksi on epävarmuutta siitä, kuinka suuri osa on toiminnallista ja mitä toiminnallisuudella missäkin yhteydessä tarkoitetaan, muuttaa tulkintaa."

        Osaatko kertoa syyn, miksi toiminnallisen genomin osuuden noustessa, siitä tulee evoluutiolle tuhoavaa?


      • Anonyymi
        Anonyymi kirjoitti:

        "Lisäksi on epävarmuutta siitä, kuinka suuri osa on toiminnallista ja mitä toiminnallisuudella missäkin yhteydessä tarkoitetaan, muuttaa tulkintaa."

        Osaatko kertoa syyn, miksi toiminnallisen genomin osuuden noustessa, siitä tulee evoluutiolle tuhoavaa?

        Olen lukenut kyseisen tutkimuksen. Kerrohan sinä ihan oma tulkintasi.


      • Anonyymi
        Anonyymi kirjoitti:

        Olen lukenut kyseisen tutkimuksen. Kerrohan sinä ihan oma tulkintasi.

        En ole lukenut tutkimusta.

        Jos suurin osa tai jopa koko genomi on toiminnallista, niin valtaosan mutaatioista täytyy olla vahingollisia.


      • Anonyymi
        Anonyymi kirjoitti:

        En ole lukenut tutkimusta.

        Jos suurin osa tai jopa koko genomi on toiminnallista, niin valtaosan mutaatioista täytyy olla vahingollisia.

        Koko genomi ei missään tapauksessa ole toiminnallista. Kymmenisen prosenttia genomistamme on ikivanhojen virusten jäänteitä. Tuskin ne ovat toimivia.

        Valtaosa mutaatioista on neutraaleja tai vahingollisia. Tämä on kauan tiedossa ollut paradigma.
        Kyse on siitä, että vahigolliset mutaatiot heikentävät pääsääntöisesti kelpoisuutta ja siksi luonnonvalinta karsii niitä tehokkaasti.
        Neutraalit ajautuvat ilman suuntaa, eivätkä vaikuta kelpoisuuteen.
        Vaikka hyödyllisiä olisi hyvinkin vähöän, niillä on taipumus lisääntyä populaatiossa.

        Tämä on aivan evoluutioteorian perusasioita.


      • Anonyymi
        Anonyymi kirjoitti:

        Koko genomi ei missään tapauksessa ole toiminnallista. Kymmenisen prosenttia genomistamme on ikivanhojen virusten jäänteitä. Tuskin ne ovat toimivia.

        Valtaosa mutaatioista on neutraaleja tai vahingollisia. Tämä on kauan tiedossa ollut paradigma.
        Kyse on siitä, että vahigolliset mutaatiot heikentävät pääsääntöisesti kelpoisuutta ja siksi luonnonvalinta karsii niitä tehokkaasti.
        Neutraalit ajautuvat ilman suuntaa, eivätkä vaikuta kelpoisuuteen.
        Vaikka hyödyllisiä olisi hyvinkin vähöän, niillä on taipumus lisääntyä populaatiossa.

        Tämä on aivan evoluutioteorian perusasioita.

        "Koko genomi ei missään tapauksessa ole toiminnallista. Kymmenisen prosenttia genomistamme on ikivanhojen virusten jäänteitä. Tuskin ne ovat toimivia."

        Genomin toiminnallisuus ylittää 100%.

        "Valtaosa mutaatioista on neutraaleja tai vahingollisia. Tämä on kauan tiedossa ollut paradigma."

        Oli tiedossa ENCODE: deen. asti. Sen jälkeen ainuttakaan mutaatioita ei voida pitää täysin neutraalina vaan valtaosaa on pidettävä vahingollisena.

        "Kyse on siitä, että vahigolliset mutaatiot heikentävät pääsääntöisesti kelpoisuutta ja siksi luonnonvalinta karsii niitä tehokkaasti."

        Kaikilla valinnan kohteilla on vahingollisia mutaatioita, joten valinta ei voi poistaa niitä.

        "Neutraalit ajautuvat ilman suuntaa, eivätkä vaikuta kelpoisuuteen.
        Vaikka hyödyllisiä olisi hyvinkin vähöän, niillä on taipumus lisääntyä populaatiossa."

        Ongelma on, että neutraaleita mutaatioita ei ole.


      • Anonyymi kirjoitti:

        "Koko genomi ei missään tapauksessa ole toiminnallista. Kymmenisen prosenttia genomistamme on ikivanhojen virusten jäänteitä. Tuskin ne ovat toimivia."

        Genomin toiminnallisuus ylittää 100%.

        "Valtaosa mutaatioista on neutraaleja tai vahingollisia. Tämä on kauan tiedossa ollut paradigma."

        Oli tiedossa ENCODE: deen. asti. Sen jälkeen ainuttakaan mutaatioita ei voida pitää täysin neutraalina vaan valtaosaa on pidettävä vahingollisena.

        "Kyse on siitä, että vahigolliset mutaatiot heikentävät pääsääntöisesti kelpoisuutta ja siksi luonnonvalinta karsii niitä tehokkaasti."

        Kaikilla valinnan kohteilla on vahingollisia mutaatioita, joten valinta ei voi poistaa niitä.

        "Neutraalit ajautuvat ilman suuntaa, eivätkä vaikuta kelpoisuuteen.
        Vaikka hyödyllisiä olisi hyvinkin vähöän, niillä on taipumus lisääntyä populaatiossa."

        Ongelma on, että neutraaleita mutaatioita ei ole.

        "Ongelma on, että neutraaleita mutaatioita ei ole."

        1. Miten määrittelet neutraalin mutaation?
        2. Jos mutaatio vaikuttaa esimerkiksi silmien väriin siten, että kahden ruskeasilmäisen afron lapsesta tuleekin sinisilmäinen, onko se haitallinen vai hyödyllinen vai olisiko ihan vain neutraali?


      • Anonyymi
        Anonyymi kirjoitti:

        "Koko genomi ei missään tapauksessa ole toiminnallista. Kymmenisen prosenttia genomistamme on ikivanhojen virusten jäänteitä. Tuskin ne ovat toimivia."

        Genomin toiminnallisuus ylittää 100%.

        "Valtaosa mutaatioista on neutraaleja tai vahingollisia. Tämä on kauan tiedossa ollut paradigma."

        Oli tiedossa ENCODE: deen. asti. Sen jälkeen ainuttakaan mutaatioita ei voida pitää täysin neutraalina vaan valtaosaa on pidettävä vahingollisena.

        "Kyse on siitä, että vahigolliset mutaatiot heikentävät pääsääntöisesti kelpoisuutta ja siksi luonnonvalinta karsii niitä tehokkaasti."

        Kaikilla valinnan kohteilla on vahingollisia mutaatioita, joten valinta ei voi poistaa niitä.

        "Neutraalit ajautuvat ilman suuntaa, eivätkä vaikuta kelpoisuuteen.
        Vaikka hyödyllisiä olisi hyvinkin vähöän, niillä on taipumus lisääntyä populaatiossa."

        Ongelma on, että neutraaleita mutaatioita ei ole.

        "Kaikilla valinnan kohteilla on vahingollisia mutaatioita, joten valinta ei voi poistaa niitä."

        Ei kukaan ole väittänyt, että valinta poistaisi kerralla kaikki vahingolliset mutaatiot. Jos vahingollisella mutaatiolla on edes parin prosentin negatiivinen vaikutus lisääntymiseen, se vähenee populaatiossa.

        Ethän sinä tunnu ymäärtävän edes alkeita. Väitteesi ovat suoraan sanoen typeriä.

        "Ongelma on, että neutraaleita mutaatioita ei ole."
        Laitahan linkki tutkmukseen, missä noin on todettu. Jos mutaatio osuu esimerkiksi siihen 10 %:iin, joka on retrovistusten jäämiä genimissamme, on se erittäin suurella todennäköisyydellä neutraali. Muitakin mahdollisuuksia on paljon.


      • Anonyymi kirjoitti:

        "Kaikilla valinnan kohteilla on vahingollisia mutaatioita, joten valinta ei voi poistaa niitä."

        Ei kukaan ole väittänyt, että valinta poistaisi kerralla kaikki vahingolliset mutaatiot. Jos vahingollisella mutaatiolla on edes parin prosentin negatiivinen vaikutus lisääntymiseen, se vähenee populaatiossa.

        Ethän sinä tunnu ymäärtävän edes alkeita. Väitteesi ovat suoraan sanoen typeriä.

        "Ongelma on, että neutraaleita mutaatioita ei ole."
        Laitahan linkki tutkmukseen, missä noin on todettu. Jos mutaatio osuu esimerkiksi siihen 10 %:iin, joka on retrovistusten jäämiä genimissamme, on se erittäin suurella todennäköisyydellä neutraali. Muitakin mahdollisuuksia on paljon.

        "Ethän sinä tunnu ymäärtävän edes alkeita. Väitteesi ovat suoraan sanoen typeriä."

        Jep. Onhan kyseessä jeesuksenmorsian.


      • Anonyymi
        marsunkiillottaja kirjoitti:

        "Ongelma on, että neutraaleita mutaatioita ei ole."

        1. Miten määrittelet neutraalin mutaation?
        2. Jos mutaatio vaikuttaa esimerkiksi silmien väriin siten, että kahden ruskeasilmäisen afron lapsesta tuleekin sinisilmäinen, onko se haitallinen vai hyödyllinen vai olisiko ihan vain neutraali?

        #1. Miten määrittelet neutraalin mutaation?#

        Määritelmä on sidoksissa siihen, onko genomi toimivaa, vai roska-DNA:ta.

        ….."Ne (tutkimusten löydöt) osoittavat, että ihmisen genomi on huomattavasti odotettua monimutkaisempi ja että lähes koko genomi käännetään - suurin osa kahteen suuntaan. He päättelivät, että suurin osa nukleotideistä ei ole ainostaan toiminnallista, vaan monitoiminnallisia, sillä niillä on useita tehtäviä. Tämä tarkoittaa, että genomin toiminnallisuus ylittää 100% (molemmat DNA-säikeet ovat pääosiltaan toiminnallisia). Tämän perusteella ainuttakaan mutaatiota ei voida pitää täysin neutraalina vaan valtaosaa on pidettävä vahingollisena....." (lainaus Sanfordin kirjasta, Eliömaailma rappeutuu).

        "Lynch väittää lähes koko ihmisgenomin olevan roska-DNA:ta, jonka takia lähes kaikki mutaatiot ovat harmittomia" (lainaus samasta kirjasta)

        #2. Jos mutaatio vaikuttaa esimerkiksi silmien väriin siten, että kahden ruskeasilmäisen afron lapsesta tuleekin sinisilmäinen, onko se haitallinen vai hyödyllinen vai olisiko ihan vain neutraali?#

        "Vaikka mutaatiot voivat vahingoittaa geenejä niin, että se vaikuttaa eloonjäämiseen, niin onneksi suurin osa niistä on selvitymisen kannalta "neutraaleja", kuten sinisimäisyysmutaatiokin näyttäisi olevan. Kun silmässä on vähemmän melaniinia, silmän verkkokalvon läpi pääsee enemmän valoa. Sinisilmäiset väittävät usein olevansa herkempiä valolle, joten sinisilmäisyydestä voi olla joitakin haittoja alueilla, joilla auringon valo on voimakasta."

        www.creationmagazine.com/creation/2018_volume_40_issue_3?pg=26#pg26


      • Anonyymi
        Anonyymi kirjoitti:

        "Kaikilla valinnan kohteilla on vahingollisia mutaatioita, joten valinta ei voi poistaa niitä."

        Ei kukaan ole väittänyt, että valinta poistaisi kerralla kaikki vahingolliset mutaatiot. Jos vahingollisella mutaatiolla on edes parin prosentin negatiivinen vaikutus lisääntymiseen, se vähenee populaatiossa.

        Ethän sinä tunnu ymäärtävän edes alkeita. Väitteesi ovat suoraan sanoen typeriä.

        "Ongelma on, että neutraaleita mutaatioita ei ole."
        Laitahan linkki tutkmukseen, missä noin on todettu. Jos mutaatio osuu esimerkiksi siihen 10 %:iin, joka on retrovistusten jäämiä genimissamme, on se erittäin suurella todennäköisyydellä neutraali. Muitakin mahdollisuuksia on paljon.

        #Ei kukaan ole väittänyt, että valinta poistaisi kerralla kaikki vahingolliset mutaatiot.#

        Eikö? " Kuinka teoreetikoiden on mahdollista perustella evolutiivista prosessia? Se tapahtuu siten, että kaikki mutaatiot lähes neutraalilla alueella jätetään huomiotta lukemalla ne täysin neutraaleiksi. Sen jälkeen oletetaan luonnonvalinnan karsivan sen vasemmalla puolella olevat mutaatiot täydellisesti. Kun tällä tavoin kaikki negatiiviset mutaatiot eliminoidaan, teoreetikot ovat vapaita väittämään, ettei ole merkitystä sillä kuinka harvinaisia positiiviset mutaatiot (oikealla puolella) ovat, sillä kehittymiselle on riittävästi aikaa ja valintavoimaa...." (lainaus kirjasta Eliömaailma rappeutuu)


      • Anonyymi
        Anonyymi kirjoitti:

        #Ei kukaan ole väittänyt, että valinta poistaisi kerralla kaikki vahingolliset mutaatiot.#

        Eikö? " Kuinka teoreetikoiden on mahdollista perustella evolutiivista prosessia? Se tapahtuu siten, että kaikki mutaatiot lähes neutraalilla alueella jätetään huomiotta lukemalla ne täysin neutraaleiksi. Sen jälkeen oletetaan luonnonvalinnan karsivan sen vasemmalla puolella olevat mutaatiot täydellisesti. Kun tällä tavoin kaikki negatiiviset mutaatiot eliminoidaan, teoreetikot ovat vapaita väittämään, ettei ole merkitystä sillä kuinka harvinaisia positiiviset mutaatiot (oikealla puolella) ovat, sillä kehittymiselle on riittävästi aikaa ja valintavoimaa...." (lainaus kirjasta Eliömaailma rappeutuu)

        "Sen jälkeen oletetaan luonnonvalinnan karsivan sen vasemmalla puolella olevat mutaatiot täydellisesti. "

        Taas tuli kretulla rakennetuksi oikein kunnolla pöllyävä olkiukko. Ei kukaan evoluutiobiologi ole tuollaista väittänyt. Tunnetaan lukuisia negatiivisia mutaatioita, joita luonnonvalinta ei poista tai poistaa huonosti. Väitteesi on hölmö.

        Joko olet totaalisen pihalla tai vääristelet tahallasi uskoen olevasi hauska. Mutta kun käyttämäsi lähteet ovat luokkaa "creationmagazine" ja luetun ymmärtäminen esikoululaisen tasolla, niin voihan sitä tuollainenkin käsitys kai muodostua.
        En tiedä kummasta enemmän on kyse, mutta uskottavuutesi on tuollaisten kommentien jälkeen pyöreä nolla.


      • Anonyymi
        Anonyymi kirjoitti:

        #Ei kukaan ole väittänyt, että valinta poistaisi kerralla kaikki vahingolliset mutaatiot.#

        Eikö? " Kuinka teoreetikoiden on mahdollista perustella evolutiivista prosessia? Se tapahtuu siten, että kaikki mutaatiot lähes neutraalilla alueella jätetään huomiotta lukemalla ne täysin neutraaleiksi. Sen jälkeen oletetaan luonnonvalinnan karsivan sen vasemmalla puolella olevat mutaatiot täydellisesti. Kun tällä tavoin kaikki negatiiviset mutaatiot eliminoidaan, teoreetikot ovat vapaita väittämään, ettei ole merkitystä sillä kuinka harvinaisia positiiviset mutaatiot (oikealla puolella) ovat, sillä kehittymiselle on riittävästi aikaa ja valintavoimaa...." (lainaus kirjasta Eliömaailma rappeutuu)

        "Kun tällä tavoin kaikki negatiiviset mutaatiot eliminoidaan, teoreetikot ovat vapaita väittämään, ettei ole merkitystä sillä kuinka harvinaisia positiiviset mutaatiot (oikealla puolella) ovat, sillä kehittymiselle on riittävästi aikaa ja valintavoimaa...."
        (lainaus kirjasta Eliömaailma rappeutuu)

        Jos Sanford on tosiaan kirjoittanut tuollaista, en yhtään ihmettele, miksi ammattitutkijat eivät ole ottaneet kirjaa vakavissaan.


      • Anonyymi
        Anonyymi kirjoitti:

        #1. Miten määrittelet neutraalin mutaation?#

        Määritelmä on sidoksissa siihen, onko genomi toimivaa, vai roska-DNA:ta.

        ….."Ne (tutkimusten löydöt) osoittavat, että ihmisen genomi on huomattavasti odotettua monimutkaisempi ja että lähes koko genomi käännetään - suurin osa kahteen suuntaan. He päättelivät, että suurin osa nukleotideistä ei ole ainostaan toiminnallista, vaan monitoiminnallisia, sillä niillä on useita tehtäviä. Tämä tarkoittaa, että genomin toiminnallisuus ylittää 100% (molemmat DNA-säikeet ovat pääosiltaan toiminnallisia). Tämän perusteella ainuttakaan mutaatiota ei voida pitää täysin neutraalina vaan valtaosaa on pidettävä vahingollisena....." (lainaus Sanfordin kirjasta, Eliömaailma rappeutuu).

        "Lynch väittää lähes koko ihmisgenomin olevan roska-DNA:ta, jonka takia lähes kaikki mutaatiot ovat harmittomia" (lainaus samasta kirjasta)

        #2. Jos mutaatio vaikuttaa esimerkiksi silmien väriin siten, että kahden ruskeasilmäisen afron lapsesta tuleekin sinisilmäinen, onko se haitallinen vai hyödyllinen vai olisiko ihan vain neutraali?#

        "Vaikka mutaatiot voivat vahingoittaa geenejä niin, että se vaikuttaa eloonjäämiseen, niin onneksi suurin osa niistä on selvitymisen kannalta "neutraaleja", kuten sinisimäisyysmutaatiokin näyttäisi olevan. Kun silmässä on vähemmän melaniinia, silmän verkkokalvon läpi pääsee enemmän valoa. Sinisilmäiset väittävät usein olevansa herkempiä valolle, joten sinisilmäisyydestä voi olla joitakin haittoja alueilla, joilla auringon valo on voimakasta."

        www.creationmagazine.com/creation/2018_volume_40_issue_3?pg=26#pg26

        Jos käytät lähteenäsi kirjaa nimeltä Eliömaailma rappeutuu, jo ennakkoasetelma on virheellinen. Mitään rappeutumista ei ole osoitettu tapahtuvan muualla kuin kreationistien märissä unissa.

        "Sinisilmäiset väittävät usein olevansa herkempiä valolle, joten sinisilmäisyydestä voi olla joitakin haittoja alueilla, joilla auringon valo on voimakasta."

        Mistä tahansa ominaisuudesta voi olla joissakin oloissa haittaa ja toisissa hyötyä. Joten entä sitten?

        - Knark5


      • Anonyymi
        Anonyymi kirjoitti:

        Jos käytät lähteenäsi kirjaa nimeltä Eliömaailma rappeutuu, jo ennakkoasetelma on virheellinen. Mitään rappeutumista ei ole osoitettu tapahtuvan muualla kuin kreationistien märissä unissa.

        "Sinisilmäiset väittävät usein olevansa herkempiä valolle, joten sinisilmäisyydestä voi olla joitakin haittoja alueilla, joilla auringon valo on voimakasta."

        Mistä tahansa ominaisuudesta voi olla joissakin oloissa haittaa ja toisissa hyötyä. Joten entä sitten?

        - Knark5

        Tummaihoisuus altistaa pohjoisessa D-vitamiinin puutokselle ja vaaleaihoisuus etelässä palamiselle ja sitä kautta melanoomalle.
        Väitse sinisilmäisyydestä on yksiselitteisesti hölmö.


      • Anonyymi
        Anonyymi kirjoitti:

        "Sen jälkeen oletetaan luonnonvalinnan karsivan sen vasemmalla puolella olevat mutaatiot täydellisesti. "

        Taas tuli kretulla rakennetuksi oikein kunnolla pöllyävä olkiukko. Ei kukaan evoluutiobiologi ole tuollaista väittänyt. Tunnetaan lukuisia negatiivisia mutaatioita, joita luonnonvalinta ei poista tai poistaa huonosti. Väitteesi on hölmö.

        Joko olet totaalisen pihalla tai vääristelet tahallasi uskoen olevasi hauska. Mutta kun käyttämäsi lähteet ovat luokkaa "creationmagazine" ja luetun ymmärtäminen esikoululaisen tasolla, niin voihan sitä tuollainenkin käsitys kai muodostua.
        En tiedä kummasta enemmän on kyse, mutta uskottavuutesi on tuollaisten kommentien jälkeen pyöreä nolla.

        "Ei kukaan evoluutiobiologi ole tuollaista väittänyt. Tunnetaan lukuisia negatiivisia mutaatioita, joita luonnonvalinta ei poista tai poistaa huonosti. Väitteesi on hölmö. "

        Etkö itse huomaa ongelmaa, jos negatiiviset mutaatiot jää poistamatta, niin ne lisääntyvät ja positiivisia ei ole. Mikä on silloin kehityksen suunta? Huomaa myös, että milloin on kyseessä teoreettikkojen näkemys ja milloin todellisuus.

        Sanford käsitteli tässä Kimuraa. Lainaukseni alusta jätin pois tämän: Kimura ei sisältänyt kaavioonsa lainkaan mutaatioita, jotka ovat nollalinjan oikealla puolella. Hän ilmeisesti ajatteli hyödyllisten mutaatioiden olevan niin harvinaisia, että ne voidaan jättää tarkastelun ulkopuolelle. Tämän mutaatiojakautuman perusteella voidaan hyvin kysyä: kuinka teoreettikkojen….

        Jonkinlaisen käsityksen kuvasta saa tästä Leisolan esityksestä, kohdasta 26.45 eteenpäin.

        https://www.youtube.com/watch?v=kLY9e4kc7Ns


      • Anonyymi
        Anonyymi kirjoitti:

        "Ei kukaan evoluutiobiologi ole tuollaista väittänyt. Tunnetaan lukuisia negatiivisia mutaatioita, joita luonnonvalinta ei poista tai poistaa huonosti. Väitteesi on hölmö. "

        Etkö itse huomaa ongelmaa, jos negatiiviset mutaatiot jää poistamatta, niin ne lisääntyvät ja positiivisia ei ole. Mikä on silloin kehityksen suunta? Huomaa myös, että milloin on kyseessä teoreettikkojen näkemys ja milloin todellisuus.

        Sanford käsitteli tässä Kimuraa. Lainaukseni alusta jätin pois tämän: Kimura ei sisältänyt kaavioonsa lainkaan mutaatioita, jotka ovat nollalinjan oikealla puolella. Hän ilmeisesti ajatteli hyödyllisten mutaatioiden olevan niin harvinaisia, että ne voidaan jättää tarkastelun ulkopuolelle. Tämän mutaatiojakautuman perusteella voidaan hyvin kysyä: kuinka teoreettikkojen….

        Jonkinlaisen käsityksen kuvasta saa tästä Leisolan esityksestä, kohdasta 26.45 eteenpäin.

        https://www.youtube.com/watch?v=kLY9e4kc7Ns

        Sanfordia on kritisoitu nimen omaan Kimuran väärästä lainaamisesta.
        Ja kuka sanoo, ettei hyödyllisiä mutaatioita muka ole. Tietenkin on.

        Osa negatiivisista mutaatioista jää evoluutiobiologien mukaan poistumatta (tosin kuin väitit vaatimukseksi), mutta niillä ei ole myöskään mekanismia lisääntyä populaatiossa. Tällöin ne ajelehtivat ilman suuntaa ja tilastollisestikin pysyvät määrältään vähäisinä (esim. verenvuototauti) ja ovat alttiita häviämiselle.
        Psotiiviset mutaatiot taas lisäävät kelpoisuutta ja runsastuvat sitä kautta populaatiossa.

        Sanfordin esitys on alusta loppuun tarkoitukhakuinen. Hänellä on ollut ensin mielessään lopputulos ja sen jälkeen haetaan arvot ( mm. Kimura) niin, että lopputulokseen päästään.

        Lainaamasi kirja on jo 15 vuotta vanha. Tiedemaailma ei ole ottanut stä tosissaan alussakaan, jolloin se vielä herätti tieteellistä keskustelua. Nyt se on jo kiinnostavuudeltaan menneen talven lumia ja kuuluu enää kreationistien lukemistoon.


      • Anonyymi
        Anonyymi kirjoitti:

        Sanfordia on kritisoitu nimen omaan Kimuran väärästä lainaamisesta.
        Ja kuka sanoo, ettei hyödyllisiä mutaatioita muka ole. Tietenkin on.

        Osa negatiivisista mutaatioista jää evoluutiobiologien mukaan poistumatta (tosin kuin väitit vaatimukseksi), mutta niillä ei ole myöskään mekanismia lisääntyä populaatiossa. Tällöin ne ajelehtivat ilman suuntaa ja tilastollisestikin pysyvät määrältään vähäisinä (esim. verenvuototauti) ja ovat alttiita häviämiselle.
        Psotiiviset mutaatiot taas lisäävät kelpoisuutta ja runsastuvat sitä kautta populaatiossa.

        Sanfordin esitys on alusta loppuun tarkoitukhakuinen. Hänellä on ollut ensin mielessään lopputulos ja sen jälkeen haetaan arvot ( mm. Kimura) niin, että lopputulokseen päästään.

        Lainaamasi kirja on jo 15 vuotta vanha. Tiedemaailma ei ole ottanut stä tosissaan alussakaan, jolloin se vielä herätti tieteellistä keskustelua. Nyt se on jo kiinnostavuudeltaan menneen talven lumia ja kuuluu enää kreationistien lukemistoon.

        "Sanfordia on kritisoitu nimen omaan Kimuran väärästä lainaamisesta.
        Ja kuka sanoo, ettei hyödyllisiä mutaatioita muka ole. Tietenkin on. "

        Ei ainakaan Sanford, hänhän lisäsi ne Kimuran kaavioon, täydentääkseen sen.

        "Osa negatiivisista mutaatioista jää evoluutiobiologien mukaan poistumatta (tosin kuin väitit vaatimukseksi), mutta niillä ei ole myöskään mekanismia lisääntyä populaatiossa. Tällöin ne ajelehtivat ilman suuntaa ja tilastollisestikin pysyvät määrältään vähäisinä (esim. verenvuototauti) ja ovat alttiita häviämiselle.
        Psotiiviset mutaatiot taas lisäävät kelpoisuutta ja runsastuvat sitä kautta populaatiossa."

        Jatketaampa Sanfordin lainausta: "....aikaa ja valintavoimaa. Tämä perustelu on kuitenkin kaikilta osiltaan väärä ja kuten pian havaitsemme. Laatikossa olevia mutaatioita ei voida jättää huomioitta, valinta ei voi poistaa kaikkia laatikon vasemmalle puolelle jääviä, eikä aika tai valintavoima riitä valitsemaan äärimmäisen harvinaisia lähes neutraalin alueen oikealla puolelle kenties ilmaantuvia hyödyllisiä mutaatioita."

        Jos sukusolua kohti tapahtuu 40 uutta proteiinien koodaamiseen vaikuttamatonta mutaatiota, tapahtuu vastasyntynyttä kohden 80 uutta mutaatiota. Lynch tunnustaa, että nämä eivät ole enimmäkseen neutraaleja vaan hienokseltaan vahingollisia. (Saford, Lynchin 2010 tekemästä tutkimuksesta) Siis hienokseltaan vahingolliset mutaatiot lisääntyvät.

        "Sanfordin esitys on alusta loppuun tarkoitukhakuinen. Hänellä on ollut ensin mielessään lopputulos ja sen jälkeen haetaan arvot ( mm. Kimura) niin, että lopputulokseen päästään."

        Miksi Sanford lisäsi hyödylliset mutaatiot Kimuran esitykseen? Mikä oli Kimuran lopputulos ja mikä Sanfordin?

        "Lainaamasi kirja on jo 15 vuotta vanha. Tiedemaailma ei ole ottanut stä tosissaan alussakaan, jolloin se vielä herätti tieteellistä keskustelua. Nyt se on jo kiinnostavuudeltaan menneen talven lumia ja kuuluu enää kreationistien lukemistoon. "

        Kirjan lukuihin on tehty päivityksiä. Viimeisimmät ovat vuodelta 2014.


      • Anonyymi kirjoitti:

        "Sanfordia on kritisoitu nimen omaan Kimuran väärästä lainaamisesta.
        Ja kuka sanoo, ettei hyödyllisiä mutaatioita muka ole. Tietenkin on. "

        Ei ainakaan Sanford, hänhän lisäsi ne Kimuran kaavioon, täydentääkseen sen.

        "Osa negatiivisista mutaatioista jää evoluutiobiologien mukaan poistumatta (tosin kuin väitit vaatimukseksi), mutta niillä ei ole myöskään mekanismia lisääntyä populaatiossa. Tällöin ne ajelehtivat ilman suuntaa ja tilastollisestikin pysyvät määrältään vähäisinä (esim. verenvuototauti) ja ovat alttiita häviämiselle.
        Psotiiviset mutaatiot taas lisäävät kelpoisuutta ja runsastuvat sitä kautta populaatiossa."

        Jatketaampa Sanfordin lainausta: "....aikaa ja valintavoimaa. Tämä perustelu on kuitenkin kaikilta osiltaan väärä ja kuten pian havaitsemme. Laatikossa olevia mutaatioita ei voida jättää huomioitta, valinta ei voi poistaa kaikkia laatikon vasemmalle puolelle jääviä, eikä aika tai valintavoima riitä valitsemaan äärimmäisen harvinaisia lähes neutraalin alueen oikealla puolelle kenties ilmaantuvia hyödyllisiä mutaatioita."

        Jos sukusolua kohti tapahtuu 40 uutta proteiinien koodaamiseen vaikuttamatonta mutaatiota, tapahtuu vastasyntynyttä kohden 80 uutta mutaatiota. Lynch tunnustaa, että nämä eivät ole enimmäkseen neutraaleja vaan hienokseltaan vahingollisia. (Saford, Lynchin 2010 tekemästä tutkimuksesta) Siis hienokseltaan vahingolliset mutaatiot lisääntyvät.

        "Sanfordin esitys on alusta loppuun tarkoitukhakuinen. Hänellä on ollut ensin mielessään lopputulos ja sen jälkeen haetaan arvot ( mm. Kimura) niin, että lopputulokseen päästään."

        Miksi Sanford lisäsi hyödylliset mutaatiot Kimuran esitykseen? Mikä oli Kimuran lopputulos ja mikä Sanfordin?

        "Lainaamasi kirja on jo 15 vuotta vanha. Tiedemaailma ei ole ottanut stä tosissaan alussakaan, jolloin se vielä herätti tieteellistä keskustelua. Nyt se on jo kiinnostavuudeltaan menneen talven lumia ja kuuluu enää kreationistien lukemistoon. "

        Kirjan lukuihin on tehty päivityksiä. Viimeisimmät ovat vuodelta 2014.

        Tiedeyhteisö ei tietääkseni ole ottanut kirjaa vakavasti. Jos se on havaittu perusteiltaan heikoksi, päivitykset eivät asiaa muuta. Harvoin kai kirjat muutenkaan muuttavat tieteellistä ajattelua. Varsinainen keskustelu käydään ainakin luonnontieteissä muilla foorumeilla, kuten tieteellisissä julkaisuissa.

        Sanford on omian väitteensä mukaan ollut ateisti, sitten teistisen evoluution kannattaja, sitten vanhan Maan kreationismin ja ID:n kannattaja. Myöhemmin on nyt kääntynyt nuorebn Maan kreationismiin ja väittänyt Maan olevan korkeintaan 100 000 vuotta vanha. Sanford on sillä väitteellään hypännyt oman tieteellisen pätevyysalueensa ulkopuolelle ja väittänyt useamman tieteenalan tutkijoiden paradigmaa vääräksi.
        En suuremmin ihmettele, ettei tiedemaailma ota häntä enää tosissaan, on sitten päivityksiä tai ei. Eikä tuokaan kirja ole edes tiedemaailmalle suunnattu. Taitaa saada paremmin voita leivälleen kreationisteilta kirjoilla ja luennoimalla.


      • agnoskepo kirjoitti:

        Tiedeyhteisö ei tietääkseni ole ottanut kirjaa vakavasti. Jos se on havaittu perusteiltaan heikoksi, päivitykset eivät asiaa muuta. Harvoin kai kirjat muutenkaan muuttavat tieteellistä ajattelua. Varsinainen keskustelu käydään ainakin luonnontieteissä muilla foorumeilla, kuten tieteellisissä julkaisuissa.

        Sanford on omian väitteensä mukaan ollut ateisti, sitten teistisen evoluution kannattaja, sitten vanhan Maan kreationismin ja ID:n kannattaja. Myöhemmin on nyt kääntynyt nuorebn Maan kreationismiin ja väittänyt Maan olevan korkeintaan 100 000 vuotta vanha. Sanford on sillä väitteellään hypännyt oman tieteellisen pätevyysalueensa ulkopuolelle ja väittänyt useamman tieteenalan tutkijoiden paradigmaa vääräksi.
        En suuremmin ihmettele, ettei tiedemaailma ota häntä enää tosissaan, on sitten päivityksiä tai ei. Eikä tuokaan kirja ole edes tiedemaailmalle suunnattu. Taitaa saada paremmin voita leivälleen kreationisteilta kirjoilla ja luennoimalla.

        Ps.
        En tämän enempää viitsi sekaantua keskusteluunne, mutta Sandord taitaa olla tieteellisesti yhtä paljon menneen talven lumia kuin joskus tiedettä tehnyt ja huuhaahan hurahtanut Sheldrake.


      • agnoskepo kirjoitti:

        Ps.
        En tämän enempää viitsi sekaantua keskusteluunne, mutta Sandord taitaa olla tieteellisesti yhtä paljon menneen talven lumia kuin joskus tiedettä tehnyt ja huuhaahan hurahtanut Sheldrake.

        Sori. Piti olla Sanford. Ihmisten nimiä ei pitäisi kirjoittaa väärin, vaikka väsy painaakin.


      • Anonyymi
        agnoskepo kirjoitti:

        Tiedeyhteisö ei tietääkseni ole ottanut kirjaa vakavasti. Jos se on havaittu perusteiltaan heikoksi, päivitykset eivät asiaa muuta. Harvoin kai kirjat muutenkaan muuttavat tieteellistä ajattelua. Varsinainen keskustelu käydään ainakin luonnontieteissä muilla foorumeilla, kuten tieteellisissä julkaisuissa.

        Sanford on omian väitteensä mukaan ollut ateisti, sitten teistisen evoluution kannattaja, sitten vanhan Maan kreationismin ja ID:n kannattaja. Myöhemmin on nyt kääntynyt nuorebn Maan kreationismiin ja väittänyt Maan olevan korkeintaan 100 000 vuotta vanha. Sanford on sillä väitteellään hypännyt oman tieteellisen pätevyysalueensa ulkopuolelle ja väittänyt useamman tieteenalan tutkijoiden paradigmaa vääräksi.
        En suuremmin ihmettele, ettei tiedemaailma ota häntä enää tosissaan, on sitten päivityksiä tai ei. Eikä tuokaan kirja ole edes tiedemaailmalle suunnattu. Taitaa saada paremmin voita leivälleen kreationisteilta kirjoilla ja luennoimalla.

        Kun ihminen hurahtaa tarpeeksi pahasti, ei tutkijan koulutus ole riittävä vastamyrkky. Niitä löytyy tieteen historiasta melkoinen määrä, eikä kenelläkään heistä ole ollut hurahtamisensa jälkeen tieteelle juuri mitään annettavaa.

        Yleensä tämä tapahtuu eläkkeellä tai ainakin lähellä eläkeikää. Sanfordkin jäi professorin virastaan eläkkeelle jo yli 20 vuotta sitten. Nuoren Maan kreationistiksi Sanford kääntyi vasta eläkkeellä ollessaan kannatettuaan vielä -90 luvun alussa teististä evoluutiota ja sen jälkeen ID oppia ja vanhan Maan kreationismia. Nyt keskustelun aiheena oleva kirja on kirjoitettu hänen oltuaan jo 7 vuotta eläkkeellä ja viitisen vuotta nuoren Maan kreationisti.

        Epäilen taustalla olevan jonkinlaisen turhautumisen tieteellisen uran loputtua. Kun luentoja ei enää ole, haetaan seurakunta, jolle saarnata totuuksia. Myös iän myötä tulevien sairauksien tuoma kognitiivisten kykyjen aleneminen saattaa joidenkin kohdalla heikentää kriittistä ajattelua.

        Ad Hominem selvästikin. Mutta näissä tapauksissa jäljet johtavat niin usein sylttytehtaalle, että jo esimerkkien takia uskottavuus karisee tuolla karrieerilla nopeasti. Ei ole alan tutkijoille yllätys, että tutkimuksia voidaan tulkita paljonkin eri tavoin, kunhan joitain oletusarvoja sopivasti viilaillaan. Ja jos tutkijan lähtökohta on nuoren Maan kreationismi, voiko häneltä objektiivisuutta edes odottaa?


      • Anonyymi
        agnoskepo kirjoitti:

        Tiedeyhteisö ei tietääkseni ole ottanut kirjaa vakavasti. Jos se on havaittu perusteiltaan heikoksi, päivitykset eivät asiaa muuta. Harvoin kai kirjat muutenkaan muuttavat tieteellistä ajattelua. Varsinainen keskustelu käydään ainakin luonnontieteissä muilla foorumeilla, kuten tieteellisissä julkaisuissa.

        Sanford on omian väitteensä mukaan ollut ateisti, sitten teistisen evoluution kannattaja, sitten vanhan Maan kreationismin ja ID:n kannattaja. Myöhemmin on nyt kääntynyt nuorebn Maan kreationismiin ja väittänyt Maan olevan korkeintaan 100 000 vuotta vanha. Sanford on sillä väitteellään hypännyt oman tieteellisen pätevyysalueensa ulkopuolelle ja väittänyt useamman tieteenalan tutkijoiden paradigmaa vääräksi.
        En suuremmin ihmettele, ettei tiedemaailma ota häntä enää tosissaan, on sitten päivityksiä tai ei. Eikä tuokaan kirja ole edes tiedemaailmalle suunnattu. Taitaa saada paremmin voita leivälleen kreationisteilta kirjoilla ja luennoimalla.

        "Tiedeyhteisö ei tietääkseni ole ottanut kirjaa vakavasti. Jos se on havaittu perusteiltaan heikoksi, päivitykset eivät asiaa muuta. Harvoin kai kirjat muutenkaan muuttavat tieteellistä ajattelua. Varsinainen keskustelu käydään ainakin luonnontieteissä muilla foorumeilla, kuten tieteellisissä julkaisuissa."

        Hyväksytkö samat "hylkäys" kriteerit kirjalle "Kelpoisimman synty"?

        Ainakin Tuulirannan mukaan:

        #Konservatiiveille Kelpoisimman synty ei ollut tervetullut suuren arvoituksen ratkaisu. Näin siksi, että evo-maailman suuria ongelmia ei ole lupa paljastaa kuin vasta sitten, kun niihin on tarjolla uskottava naturalistinen ratkaisu. Sellaisen Wagner kuvitteli löytäneensä ja siksi hätiköity paljastus: ”Moderni synteesi selitti, miten innovaatiot leviävät, mutta ei niiden alkuperää.” #


      • Anonyymi
        Anonyymi kirjoitti:

        Kun ihminen hurahtaa tarpeeksi pahasti, ei tutkijan koulutus ole riittävä vastamyrkky. Niitä löytyy tieteen historiasta melkoinen määrä, eikä kenelläkään heistä ole ollut hurahtamisensa jälkeen tieteelle juuri mitään annettavaa.

        Yleensä tämä tapahtuu eläkkeellä tai ainakin lähellä eläkeikää. Sanfordkin jäi professorin virastaan eläkkeelle jo yli 20 vuotta sitten. Nuoren Maan kreationistiksi Sanford kääntyi vasta eläkkeellä ollessaan kannatettuaan vielä -90 luvun alussa teististä evoluutiota ja sen jälkeen ID oppia ja vanhan Maan kreationismia. Nyt keskustelun aiheena oleva kirja on kirjoitettu hänen oltuaan jo 7 vuotta eläkkeellä ja viitisen vuotta nuoren Maan kreationisti.

        Epäilen taustalla olevan jonkinlaisen turhautumisen tieteellisen uran loputtua. Kun luentoja ei enää ole, haetaan seurakunta, jolle saarnata totuuksia. Myös iän myötä tulevien sairauksien tuoma kognitiivisten kykyjen aleneminen saattaa joidenkin kohdalla heikentää kriittistä ajattelua.

        Ad Hominem selvästikin. Mutta näissä tapauksissa jäljet johtavat niin usein sylttytehtaalle, että jo esimerkkien takia uskottavuus karisee tuolla karrieerilla nopeasti. Ei ole alan tutkijoille yllätys, että tutkimuksia voidaan tulkita paljonkin eri tavoin, kunhan joitain oletusarvoja sopivasti viilaillaan. Ja jos tutkijan lähtökohta on nuoren Maan kreationismi, voiko häneltä objektiivisuutta edes odottaa?

        #Kun ihminen hurahtaa tarpeeksi pahasti, ei tutkijan koulutus ole riittävä vastamyrkky. Niitä löytyy tieteen historiasta melkoinen määrä, eikä kenelläkään heistä ole ollut hurahtamisensa jälkeen tieteelle juuri mitään annettavaa.#

        ”Mutta ehkä näiden abstraktien teorioiden ohuen ilmanalan hengittäminen liian pitkään on alkanut synnyttää hallusinaatioita ja mielikuvituksemme luomat fantasiat saavat asiat näyttämään sellaisilta millaisina me haluaisimmekin ne nähdä” (s.114)?

        Tuulirannan lainaus Behen uusimmasta kirjasta.


      • Anonyymi kirjoitti:

        "Tiedeyhteisö ei tietääkseni ole ottanut kirjaa vakavasti. Jos se on havaittu perusteiltaan heikoksi, päivitykset eivät asiaa muuta. Harvoin kai kirjat muutenkaan muuttavat tieteellistä ajattelua. Varsinainen keskustelu käydään ainakin luonnontieteissä muilla foorumeilla, kuten tieteellisissä julkaisuissa."

        Hyväksytkö samat "hylkäys" kriteerit kirjalle "Kelpoisimman synty"?

        Ainakin Tuulirannan mukaan:

        #Konservatiiveille Kelpoisimman synty ei ollut tervetullut suuren arvoituksen ratkaisu. Näin siksi, että evo-maailman suuria ongelmia ei ole lupa paljastaa kuin vasta sitten, kun niihin on tarjolla uskottava naturalistinen ratkaisu. Sellaisen Wagner kuvitteli löytäneensä ja siksi hätiköity paljastus: ”Moderni synteesi selitti, miten innovaatiot leviävät, mutta ei niiden alkuperää.” #

        En mielelläni vastaisi toisten puolesta, mutta Wagnerin kirjan pariin kertaan lukeneena ja hänen ryhmänsä töihin muutenkin tutustuneena otan kantaa.

        Tuuliranta on kreationisti, jonka kanta ei tiedeyhteisöä.
        "Konservatiiveille Kelpoisimman synty ei ollut tervetullut suuren arvoituksen ratkaisu."
        Onko kirja konservatiivien mukaan tervetullut vai ei, ei muuta sen arvoa. Ja keitähän Tuuliranta mahtaa "konservatiiveilla" tarkoittaa? Tuulirannan edustamalle uskomuspohjaiselle opille mikään evoluutioteoriaa tukeva ratkaisu ei ole tervetullut.
        Onko sinulla tieteellisempää kritiikkisä kyseistä kirjaa kohtaan?

        Wagner huippuryhmineen on erittäin arvostettu tutkija eikä se arvostus ei perustu kyseiseen kirjaan, vaan korkealaatuiseen tutkimukseen.


      • Anonyymi kirjoitti:

        "Tiedeyhteisö ei tietääkseni ole ottanut kirjaa vakavasti. Jos se on havaittu perusteiltaan heikoksi, päivitykset eivät asiaa muuta. Harvoin kai kirjat muutenkaan muuttavat tieteellistä ajattelua. Varsinainen keskustelu käydään ainakin luonnontieteissä muilla foorumeilla, kuten tieteellisissä julkaisuissa."

        Hyväksytkö samat "hylkäys" kriteerit kirjalle "Kelpoisimman synty"?

        Ainakin Tuulirannan mukaan:

        #Konservatiiveille Kelpoisimman synty ei ollut tervetullut suuren arvoituksen ratkaisu. Näin siksi, että evo-maailman suuria ongelmia ei ole lupa paljastaa kuin vasta sitten, kun niihin on tarjolla uskottava naturalistinen ratkaisu. Sellaisen Wagner kuvitteli löytäneensä ja siksi hätiköity paljastus: ”Moderni synteesi selitti, miten innovaatiot leviävät, mutta ei niiden alkuperää.” #

        "Sellaisen Wagner kuvitteli löytäneensä ja siksi hätiköity paljastus: ”Moderni synteesi selitti, miten innovaatiot leviävät, mutta ei niiden alkuperää.”"

        Tuuliranta taitaa olla väitteineen rannalla. Juuri innovaatioiden synnyn Wagnerin kirja selitti. Se selitti, miten on mahdollista se, että satunnaiset muutokset voivat kehittää toimivia innovaatioita. Yhden askeleen muutoksilla eri innovaatiot voivat muuttua rakenteeltaan suurelta osin säilyttäen samalla toiminnallisuutensa. Wagnerin omaa luonnehdintaa käyttäen toimivia innovaatioita on hypertähtitieteellinen määrä.


      • Anonyymi
        Anonyymi kirjoitti:

        "Tiedeyhteisö ei tietääkseni ole ottanut kirjaa vakavasti. Jos se on havaittu perusteiltaan heikoksi, päivitykset eivät asiaa muuta. Harvoin kai kirjat muutenkaan muuttavat tieteellistä ajattelua. Varsinainen keskustelu käydään ainakin luonnontieteissä muilla foorumeilla, kuten tieteellisissä julkaisuissa."

        Hyväksytkö samat "hylkäys" kriteerit kirjalle "Kelpoisimman synty"?

        Ainakin Tuulirannan mukaan:

        #Konservatiiveille Kelpoisimman synty ei ollut tervetullut suuren arvoituksen ratkaisu. Näin siksi, että evo-maailman suuria ongelmia ei ole lupa paljastaa kuin vasta sitten, kun niihin on tarjolla uskottava naturalistinen ratkaisu. Sellaisen Wagner kuvitteli löytäneensä ja siksi hätiköity paljastus: ”Moderni synteesi selitti, miten innovaatiot leviävät, mutta ei niiden alkuperää.” #

        "Näin siksi, että evo-maailman suuria ongelmia ei ole lupa paljastaa kuin vasta sitten, kun niihin on tarjolla uskottava naturalistinen ratkaisu. "

        Mikä on tieteessä se instanssi, joka tieteellisiä paljastuslupia jakelee tai kieltää. Väite oli tohtoritason mieheltä naiivi, olkoon sitten kysymys vaikka kirurgian tohtorista. Joko Tuulirannalla on alkanut vanhuuttaan viirata tai sitten hän valehtelee tahallaan seurakuntansa lähdekritiikin tason tuntien.

        "Mutta ehkä näiden abstraktien teorioiden ohuen ilmanalan hengittäminen liian pitkään on alkanut synnyttää hallusinaatioita ja mielikuvituksemme luomat fantasiat saavat asiat näyttämään sellaisilta millaisina me haluaisimmekin ne nähdä."

        Varsin tieteellinen väite tuokin. Taitaa taivaallisten sfäärien ilmanalan gengittäminen aiheuttaa paljon pahempia houreita lattialla sätkimisineen.


      • Anonyymi
        agnoskepo kirjoitti:

        "Sellaisen Wagner kuvitteli löytäneensä ja siksi hätiköity paljastus: ”Moderni synteesi selitti, miten innovaatiot leviävät, mutta ei niiden alkuperää.”"

        Tuuliranta taitaa olla väitteineen rannalla. Juuri innovaatioiden synnyn Wagnerin kirja selitti. Se selitti, miten on mahdollista se, että satunnaiset muutokset voivat kehittää toimivia innovaatioita. Yhden askeleen muutoksilla eri innovaatiot voivat muuttua rakenteeltaan suurelta osin säilyttäen samalla toiminnallisuutensa. Wagnerin omaa luonnehdintaa käyttäen toimivia innovaatioita on hypertähtitieteellinen määrä.

        "Tuuliranta taitaa olla väitteineen rannalla. Juuri innovaatioiden synnyn Wagnerin kirja selitti."

        Tuuliranta alla olevan linkin sisällön mukaan, sivu 30: Wagner kysyy innovaatioiden alkuperää ja tarjoaa vastaukseksi mystiikkaa: ”Vastaus on näkymättömässä kädessä, joka ohjasi maailmaa kauan ennen niiden syntymää: itseorganisaatiossa – ja vieläpä sen oudossa lajissa” (s.156). Tiedettä?

        https://luominen.fi/materiaalit/pdf/behe_darwin_devolves_poimintoja_uutuuskirjasta_darwin_taantuu.pdf


      • Anonyymi
        agnoskepo kirjoitti:

        En mielelläni vastaisi toisten puolesta, mutta Wagnerin kirjan pariin kertaan lukeneena ja hänen ryhmänsä töihin muutenkin tutustuneena otan kantaa.

        Tuuliranta on kreationisti, jonka kanta ei tiedeyhteisöä.
        "Konservatiiveille Kelpoisimman synty ei ollut tervetullut suuren arvoituksen ratkaisu."
        Onko kirja konservatiivien mukaan tervetullut vai ei, ei muuta sen arvoa. Ja keitähän Tuuliranta mahtaa "konservatiiveilla" tarkoittaa? Tuulirannan edustamalle uskomuspohjaiselle opille mikään evoluutioteoriaa tukeva ratkaisu ei ole tervetullut.
        Onko sinulla tieteellisempää kritiikkisä kyseistä kirjaa kohtaan?

        Wagner huippuryhmineen on erittäin arvostettu tutkija eikä se arvostus ei perustu kyseiseen kirjaan, vaan korkealaatuiseen tutkimukseen.

        "Onko kirja konservatiivien mukaan tervetullut vai ei, ei muuta sen arvoa. Ja keitähän Tuuliranta mahtaa "konservatiiveilla" tarkoittaa? Tuulirannan edustamalle uskomuspohjaiselle opille mikään evoluutioteoriaa tukeva ratkaisu ei ole tervetullut."

        Ehkä Tuuliranta tarkoittaa konservatiiveilla heitä: (Lontoon Royal Society järjesti syyskuussa 2016 ”kriisikongressin” New Perspectives in Evolutionary Biology. Salaisuuksien paljastajana Wagneria ei tilaisuuteen kutsuttu. Ja jo vuonna 1994 Leo Buss ja Walter Fontana kirjoittivat samannimisen pamfletin, Arrival of the Fittest, koska hekin luulivat ratkaisseensa Darwinin dilemman – aivan kuten Marc Kirschner ja John Gerhart isäntägeeneillään ja evo-devo – hypoteesillaan [ks. seur. luku].)


      • Anonyymi
        Anonyymi kirjoitti:

        Kun ihminen hurahtaa tarpeeksi pahasti, ei tutkijan koulutus ole riittävä vastamyrkky. Niitä löytyy tieteen historiasta melkoinen määrä, eikä kenelläkään heistä ole ollut hurahtamisensa jälkeen tieteelle juuri mitään annettavaa.

        Yleensä tämä tapahtuu eläkkeellä tai ainakin lähellä eläkeikää. Sanfordkin jäi professorin virastaan eläkkeelle jo yli 20 vuotta sitten. Nuoren Maan kreationistiksi Sanford kääntyi vasta eläkkeellä ollessaan kannatettuaan vielä -90 luvun alussa teististä evoluutiota ja sen jälkeen ID oppia ja vanhan Maan kreationismia. Nyt keskustelun aiheena oleva kirja on kirjoitettu hänen oltuaan jo 7 vuotta eläkkeellä ja viitisen vuotta nuoren Maan kreationisti.

        Epäilen taustalla olevan jonkinlaisen turhautumisen tieteellisen uran loputtua. Kun luentoja ei enää ole, haetaan seurakunta, jolle saarnata totuuksia. Myös iän myötä tulevien sairauksien tuoma kognitiivisten kykyjen aleneminen saattaa joidenkin kohdalla heikentää kriittistä ajattelua.

        Ad Hominem selvästikin. Mutta näissä tapauksissa jäljet johtavat niin usein sylttytehtaalle, että jo esimerkkien takia uskottavuus karisee tuolla karrieerilla nopeasti. Ei ole alan tutkijoille yllätys, että tutkimuksia voidaan tulkita paljonkin eri tavoin, kunhan joitain oletusarvoja sopivasti viilaillaan. Ja jos tutkijan lähtökohta on nuoren Maan kreationismi, voiko häneltä objektiivisuutta edes odottaa?

        Hieno kirjoitus. Vähän kuin Putin on pieni, Tai Trumpilla on nakkisormet. Informaatiosta viis. Ulkoiset seikat ratkaisevat. Monille haisee, ihan oikeasti. Haju etoo. Omaavat ihailtavan kaukoaistin. Toiset näkevät ilman kristallipalloa tapahtumia.


      • Anonyymi
        Anonyymi kirjoitti:

        Sanfordia on kritisoitu nimen omaan Kimuran väärästä lainaamisesta.
        Ja kuka sanoo, ettei hyödyllisiä mutaatioita muka ole. Tietenkin on.

        Osa negatiivisista mutaatioista jää evoluutiobiologien mukaan poistumatta (tosin kuin väitit vaatimukseksi), mutta niillä ei ole myöskään mekanismia lisääntyä populaatiossa. Tällöin ne ajelehtivat ilman suuntaa ja tilastollisestikin pysyvät määrältään vähäisinä (esim. verenvuototauti) ja ovat alttiita häviämiselle.
        Psotiiviset mutaatiot taas lisäävät kelpoisuutta ja runsastuvat sitä kautta populaatiossa.

        Sanfordin esitys on alusta loppuun tarkoitukhakuinen. Hänellä on ollut ensin mielessään lopputulos ja sen jälkeen haetaan arvot ( mm. Kimura) niin, että lopputulokseen päästään.

        Lainaamasi kirja on jo 15 vuotta vanha. Tiedemaailma ei ole ottanut stä tosissaan alussakaan, jolloin se vielä herätti tieteellistä keskustelua. Nyt se on jo kiinnostavuudeltaan menneen talven lumia ja kuuluu enää kreationistien lukemistoon.

        Kirja kumoaa koko kehitysopin. Sanford on pyytänyt kirjasta mielipiteitä kolleegoiltaan. Siis vähintään yliopistotasoisilta tutkijoilta.

        https://puheenvuoro.uusisuomi.fi/ilkkahyttinen/104067-rappeutuminen-kumoaa-kehitysopin/


      • Anonyymi kirjoitti:

        Kirja kumoaa koko kehitysopin. Sanford on pyytänyt kirjasta mielipiteitä kolleegoiltaan. Siis vähintään yliopistotasoisilta tutkijoilta.

        https://puheenvuoro.uusisuomi.fi/ilkkahyttinen/104067-rappeutuminen-kumoaa-kehitysopin/

        Odotellaan ja katsotaan. Jos kirja on niin pätevä kuin jokunen kreationisti ilmeisesti toivoo, suuri osa luonnontieteistä menee uusiksi.

        Hyttisen blogin kommenteista:

        "Evoluutioteoriaa suojelee ateistinen salaliitto ja evoluutioteoria aiheuttaa maailman sodat, nälänhädät, tsunamit, väkivallan ja raiskaukset."

        Pahimmassa tapauksessa tuonkin kirjoittaja on tosissaan.

        "”Evotiedeuskonnon” salaseuraanhan ei hyväksytä ketään evoluution kumoajaa, joten ei voida puhua vertaisarvioinneistakaan, koska kreationismin, Luoja Jumalan vertaista ei löydy mistään universumista…"

        Ja tuon.

        Hyttisen blogikirjoitus on muuten yli seitsemän vuoden takaa. Ilmeisesti kirja ei ole ainakaan toistaiseksi kumonnut kehitysoppia, kun ei siitä ole ollut kapybarankokoisia otsikoita missään.


      • Anonyymi
        marsunkiillottaja kirjoitti:

        Odotellaan ja katsotaan. Jos kirja on niin pätevä kuin jokunen kreationisti ilmeisesti toivoo, suuri osa luonnontieteistä menee uusiksi.

        Hyttisen blogin kommenteista:

        "Evoluutioteoriaa suojelee ateistinen salaliitto ja evoluutioteoria aiheuttaa maailman sodat, nälänhädät, tsunamit, väkivallan ja raiskaukset."

        Pahimmassa tapauksessa tuonkin kirjoittaja on tosissaan.

        "”Evotiedeuskonnon” salaseuraanhan ei hyväksytä ketään evoluution kumoajaa, joten ei voida puhua vertaisarvioinneistakaan, koska kreationismin, Luoja Jumalan vertaista ei löydy mistään universumista…"

        Ja tuon.

        Hyttisen blogikirjoitus on muuten yli seitsemän vuoden takaa. Ilmeisesti kirja ei ole ainakaan toistaiseksi kumonnut kehitysoppia, kun ei siitä ole ollut kapybarankokoisia otsikoita missään.

        Tuossa uudempi kirja ja sen arvostelu. Ei kuitenkaan samalta kirjoittajalta. Stanfordin kirjan kirjoittamisen ajalta ei matematiikka tai aminohappojen rakenteessa ole tapahtunut muutoksia. Ihmisen genomi on vain huonompi.

        https://luominen.fi/darwin-taantuu-uusi-dna-tiede-haastaa-evoluution-poimintoja-uutuuskirjasta-darwin-devolves


      • Anonyymi
        Anonyymi kirjoitti:

        Tuossa uudempi kirja ja sen arvostelu. Ei kuitenkaan samalta kirjoittajalta. Stanfordin kirjan kirjoittamisen ajalta ei matematiikka tai aminohappojen rakenteessa ole tapahtunut muutoksia. Ihmisen genomi on vain huonompi.

        https://luominen.fi/darwin-taantuu-uusi-dna-tiede-haastaa-evoluution-poimintoja-uutuuskirjasta-darwin-devolves

        Millä täsmällisesti määriteltävällä tavalla ihmisen genomi on nyt "huonompi" tai huonompi?

        Kontrollikysymys: onko luominen.fi tieteellinen sivusto?


    • Anonyymi

      "Ja koska evoluutiota ei siis täten tapahdu, eikä ole koskaan tapahtunutkaan, on luominen ainoa looginen johtopäätelmä."

      Montako kertaa sinulle pitää muistuttaa, että muita varteenotettavia mahdollisuuksia on vähintään kolme, uusimpien tutkimusten mukaan ehkä jopa noin kuusikymmentä? Eikö mene perille?

      "Raamattu on totta ensimmäisestä viimeiseen jakeeseen."

      Kumpi kahdesta erilaisesta luomiskertomuksesta kuvaa tapahtumien järjestyksen täsmälleen oikein?

      "Se on Jumalan Sanaa, eikä sen arvovaltaa saa väheksyä."

      Miksi sitten jatkuvasti väheksyt sitä?

      Pystykö varmasti leipätyössäsi pitäytymään opetussuunnitelmassa, niin ettei maailmankatsomuksesi vahingossakaan tunkeudu edes uskonnonopetukseen? Mikset vastaa tähän suhteellisen yksinkertaiseen kysymykseen?

      • Anonyymi

        Ei siis täten tapahdu...
        Lapsellinen väite, jolla ei ole tieteellisesti mitään perusteita.

        Laitahan nyt viimein ROT edes yksi vertaisarvioitu tutkimus, jossa todetaan, ettei evoluutiota tapahdu.

        Tai laita yksikin geologian vertaiarvioitu tutkimus, joka väittää Maan iäksi 6000 vuotta.

        Olet pelkkä uskonnollinen pelle.


    • ihmislajilla reilu 7 miljardia erilaista geneettistä variaatiota. Laji elää ja voi hyvin.

      • Anonyymi

        Mitokondrio periytyy naisen munasolussa kuten Y kromosomi miehen siittiössä. kumpikaan ei saa mistään mutaaation tilalle uutta

        Mitokondriotaudit

        https://www.duodecimlehti.fi/lehti/1992/6/duo20114

        Uusia Mitokondriotauteja tulee lisää.
        Andokrinologiset muutokset:
        Lyhytkasvuisuus
        Infertiiliys
        Viivästynyt puberteetti
        Hypoparatyreoosi
        Diabetes

        Diabetes lisääntyy hurjasti eikä luonnonvalinta sitä poista. Vähänkään populaatioteoriaa tuntevana ymmärrät taudin viimein saavuttavan kaikki jälkeläiset. Periytyvyys menee läpi koko populaation.

        Laji ei todellakaan voi hyvin vaan rappeutuu. On tulossa yhä riippuvaisemmaksi lääkkeistä.


        Taudit lisääntyvät eikä mitokondrio saa mistään uuutaa


      • Anonyymi
        Anonyymi kirjoitti:

        Mitokondrio periytyy naisen munasolussa kuten Y kromosomi miehen siittiössä. kumpikaan ei saa mistään mutaaation tilalle uutta

        Mitokondriotaudit

        https://www.duodecimlehti.fi/lehti/1992/6/duo20114

        Uusia Mitokondriotauteja tulee lisää.
        Andokrinologiset muutokset:
        Lyhytkasvuisuus
        Infertiiliys
        Viivästynyt puberteetti
        Hypoparatyreoosi
        Diabetes

        Diabetes lisääntyy hurjasti eikä luonnonvalinta sitä poista. Vähänkään populaatioteoriaa tuntevana ymmärrät taudin viimein saavuttavan kaikki jälkeläiset. Periytyvyys menee läpi koko populaation.

        Laji ei todellakaan voi hyvin vaan rappeutuu. On tulossa yhä riippuvaisemmaksi lääkkeistä.


        Taudit lisääntyvät eikä mitokondrio saa mistään uuutaa

        "Diabetes lisääntyy hurjasti eikä luonnonvalinta sitä poista. Vähänkään populaatioteoriaa tuntevana ymmärrät taudin viimein saavuttavan kaikki jälkeläiset. Periytyvyys menee läpi koko populaation.

        Laji ei todellakaan voi hyvin vaan rappeutuu. On tulossa yhä riippuvaisemmaksi lääkkeistä.

        Taudit lisääntyvät eikä mitokondrio saa mistään uuutaa"
        Edelleen, pystytkö osoittamaan, että yhdessäkään sukupuuttotapauksessa olisi primäärisyynä ollut rappeutuminen?
        Lainataanpas tässä kohdin niin kutsuttua "ateistista tiedettä", eli Ihmisen fysiologia ja anatomia-kirjaa (Nienstedt, Hänninen, Arstila, Björkqvist): "Koska luonnonvalinta turvaa jälkeläisten saamisen, se on varsin tehoton karsimaan häiriöitä tai sairauksia, jotka esiintyvät VASTA lisääntymisiän jälkeen. Niinpä esim. eturauhasen liikakasvu kohtaa lähes kaikkia vanhoja miehiä. Luonnonvalinta ei ole tätä puutetta korjannut, koska vanhat miehet eivät muutenkaan yleensä enää saa jälkeläisiä. Sama koskee valtimonkovetustautia ja useimpia syöpälajeja. Niihin sairastutaan niin myöhään, etteivät ne ole kovin merkityksellisiä luonnonvalinnan kannalta."
        Mainitsemistasi sairauksista oikeastaan steriiliys estää lisääntymisen, ja senkin kohdalla luonnonvalinta toimii karsien nämä geenit pois siinä vaiheessa, kun ne eivät enää pysty siirtymään seuraavalle sukupolvelle taudin itsensä vuoksi. Diabeteskin yleistyy oikeastaan siitä syystä, että nykyisellä sukupolvella on niin paskat elintavat. Usko pois, tätä on tutkittu paljon täällä Suomessakin yliopistoissa. Kaikkialla maailmassa diabetes ei ole vastaavalla tavalla ongelma kuin täällä Suomessa, joten taudin periytyvyys tuskin tulee ikinä läpileikkaamaan koko ihmiskuntaa, koska ulkoisilla tekijöillä on diabeteksen yhteydessä niin iso rooli. Tätä päätelmää tukee myös tutkimukset:
        "Taustaväestöä vastaavasta tyypin 1 diabeteksen esiintyvyydestä huolimatta havainto somalitaustaisten lasten erilaisesta geneettistä riskiprofiilista korostaa ulkoisten tekijöiden merkitystä diabeteksen patogeneesissä. Somaliasta ei ole olemassa yhtään tyypin 1 diabetesta koskevaa epidemiologista tutkimusta, mutta muista Afrikan maista saatujen tulosten (Sudanissa, Nigeriassa ja Algeriassa esiintyvyydeksi on saatu 2,7 - 9,5/10 000) perusteella voidaan olettaa, että taudin esiintyvyys on selvästi pienempi kuin Suomessa. Eräissä muissa maissa tehdyissä tutkimuksissa on myös havaittu, että maahanmuuttajien lapsilla tyypin 1 diabeteksen ilmaantuvuus muuttuu samankaltaiseksi kuin kohdemaassa."
        https://www.duodecimlehti.fi/duo10192


      • Anonyymi
        Anonyymi kirjoitti:

        "Diabetes lisääntyy hurjasti eikä luonnonvalinta sitä poista. Vähänkään populaatioteoriaa tuntevana ymmärrät taudin viimein saavuttavan kaikki jälkeläiset. Periytyvyys menee läpi koko populaation.

        Laji ei todellakaan voi hyvin vaan rappeutuu. On tulossa yhä riippuvaisemmaksi lääkkeistä.

        Taudit lisääntyvät eikä mitokondrio saa mistään uuutaa"
        Edelleen, pystytkö osoittamaan, että yhdessäkään sukupuuttotapauksessa olisi primäärisyynä ollut rappeutuminen?
        Lainataanpas tässä kohdin niin kutsuttua "ateistista tiedettä", eli Ihmisen fysiologia ja anatomia-kirjaa (Nienstedt, Hänninen, Arstila, Björkqvist): "Koska luonnonvalinta turvaa jälkeläisten saamisen, se on varsin tehoton karsimaan häiriöitä tai sairauksia, jotka esiintyvät VASTA lisääntymisiän jälkeen. Niinpä esim. eturauhasen liikakasvu kohtaa lähes kaikkia vanhoja miehiä. Luonnonvalinta ei ole tätä puutetta korjannut, koska vanhat miehet eivät muutenkaan yleensä enää saa jälkeläisiä. Sama koskee valtimonkovetustautia ja useimpia syöpälajeja. Niihin sairastutaan niin myöhään, etteivät ne ole kovin merkityksellisiä luonnonvalinnan kannalta."
        Mainitsemistasi sairauksista oikeastaan steriiliys estää lisääntymisen, ja senkin kohdalla luonnonvalinta toimii karsien nämä geenit pois siinä vaiheessa, kun ne eivät enää pysty siirtymään seuraavalle sukupolvelle taudin itsensä vuoksi. Diabeteskin yleistyy oikeastaan siitä syystä, että nykyisellä sukupolvella on niin paskat elintavat. Usko pois, tätä on tutkittu paljon täällä Suomessakin yliopistoissa. Kaikkialla maailmassa diabetes ei ole vastaavalla tavalla ongelma kuin täällä Suomessa, joten taudin periytyvyys tuskin tulee ikinä läpileikkaamaan koko ihmiskuntaa, koska ulkoisilla tekijöillä on diabeteksen yhteydessä niin iso rooli. Tätä päätelmää tukee myös tutkimukset:
        "Taustaväestöä vastaavasta tyypin 1 diabeteksen esiintyvyydestä huolimatta havainto somalitaustaisten lasten erilaisesta geneettistä riskiprofiilista korostaa ulkoisten tekijöiden merkitystä diabeteksen patogeneesissä. Somaliasta ei ole olemassa yhtään tyypin 1 diabetesta koskevaa epidemiologista tutkimusta, mutta muista Afrikan maista saatujen tulosten (Sudanissa, Nigeriassa ja Algeriassa esiintyvyydeksi on saatu 2,7 - 9,5/10 000) perusteella voidaan olettaa, että taudin esiintyvyys on selvästi pienempi kuin Suomessa. Eräissä muissa maissa tehdyissä tutkimuksissa on myös havaittu, että maahanmuuttajien lapsilla tyypin 1 diabeteksen ilmaantuvuus muuttuu samankaltaiseksi kuin kohdemaassa."
        https://www.duodecimlehti.fi/duo10192

        Ja taas meni Raamattullinen rappeuma -sadusta hörhöilevä hihu hiljaiseksi.


      • Anonyymi kirjoitti:

        "Diabetes lisääntyy hurjasti eikä luonnonvalinta sitä poista. Vähänkään populaatioteoriaa tuntevana ymmärrät taudin viimein saavuttavan kaikki jälkeläiset. Periytyvyys menee läpi koko populaation.

        Laji ei todellakaan voi hyvin vaan rappeutuu. On tulossa yhä riippuvaisemmaksi lääkkeistä.

        Taudit lisääntyvät eikä mitokondrio saa mistään uuutaa"
        Edelleen, pystytkö osoittamaan, että yhdessäkään sukupuuttotapauksessa olisi primäärisyynä ollut rappeutuminen?
        Lainataanpas tässä kohdin niin kutsuttua "ateistista tiedettä", eli Ihmisen fysiologia ja anatomia-kirjaa (Nienstedt, Hänninen, Arstila, Björkqvist): "Koska luonnonvalinta turvaa jälkeläisten saamisen, se on varsin tehoton karsimaan häiriöitä tai sairauksia, jotka esiintyvät VASTA lisääntymisiän jälkeen. Niinpä esim. eturauhasen liikakasvu kohtaa lähes kaikkia vanhoja miehiä. Luonnonvalinta ei ole tätä puutetta korjannut, koska vanhat miehet eivät muutenkaan yleensä enää saa jälkeläisiä. Sama koskee valtimonkovetustautia ja useimpia syöpälajeja. Niihin sairastutaan niin myöhään, etteivät ne ole kovin merkityksellisiä luonnonvalinnan kannalta."
        Mainitsemistasi sairauksista oikeastaan steriiliys estää lisääntymisen, ja senkin kohdalla luonnonvalinta toimii karsien nämä geenit pois siinä vaiheessa, kun ne eivät enää pysty siirtymään seuraavalle sukupolvelle taudin itsensä vuoksi. Diabeteskin yleistyy oikeastaan siitä syystä, että nykyisellä sukupolvella on niin paskat elintavat. Usko pois, tätä on tutkittu paljon täällä Suomessakin yliopistoissa. Kaikkialla maailmassa diabetes ei ole vastaavalla tavalla ongelma kuin täällä Suomessa, joten taudin periytyvyys tuskin tulee ikinä läpileikkaamaan koko ihmiskuntaa, koska ulkoisilla tekijöillä on diabeteksen yhteydessä niin iso rooli. Tätä päätelmää tukee myös tutkimukset:
        "Taustaväestöä vastaavasta tyypin 1 diabeteksen esiintyvyydestä huolimatta havainto somalitaustaisten lasten erilaisesta geneettistä riskiprofiilista korostaa ulkoisten tekijöiden merkitystä diabeteksen patogeneesissä. Somaliasta ei ole olemassa yhtään tyypin 1 diabetesta koskevaa epidemiologista tutkimusta, mutta muista Afrikan maista saatujen tulosten (Sudanissa, Nigeriassa ja Algeriassa esiintyvyydeksi on saatu 2,7 - 9,5/10 000) perusteella voidaan olettaa, että taudin esiintyvyys on selvästi pienempi kuin Suomessa. Eräissä muissa maissa tehdyissä tutkimuksissa on myös havaittu, että maahanmuuttajien lapsilla tyypin 1 diabeteksen ilmaantuvuus muuttuu samankaltaiseksi kuin kohdemaassa."
        https://www.duodecimlehti.fi/duo10192

        "Laji ei todellakaan voi hyvin vaan rappeutuu."

        Niinkö artikkelissa luki? Minä ymmärsin artikkelin aivan toisin.

        " Niihin sairastutaan niin myöhään, etteivät ne ole kovin merkityksellisiä luonnonvalinnan kannalta."
        Eikä myöskään populaation kelpoisuuden kannalta. Luonnonvalinta ei poista virheitä, jotka vaikuttavatn vasta lisääntymisiän jälkeen, mutta eipä niillä ole myöskään sanottavaa rappeuttavaa vaikutusta. Ei ainakan niin, että populaation säilyvyys olisi uhattuna.

        " On tulossa yhä riippuvaisemmaksi lääkkeistä."
        Emme olisi sen riippuvaisempia lääkkeistä kuin ennenkään, jos odotettavissa oleva elinikämme heiluisi 40 vuoden paikkeilla, eikä 80 vuoden. Lääkkeillä on kyetty nostamaan elinikää, mikä samalla altistaa vanhenemisen tuomille lisääntyville sairauksille. Populaation genomin rappeutumisen kanssa tälläkään ei ole mitään tekemistä.

        "Edelleen, pystytkö osoittamaan, että yhdessäkään sukupuuttotapauksessa olisi primäärisyynä ollut rappeutuminen?"

        Tuohon kysymykseen et kommentoinut mitenkään, vaikka olit lainannut sitä juttusi alkuun.


      • Anonyymi kirjoitti:

        Mitokondrio periytyy naisen munasolussa kuten Y kromosomi miehen siittiössä. kumpikaan ei saa mistään mutaaation tilalle uutta

        Mitokondriotaudit

        https://www.duodecimlehti.fi/lehti/1992/6/duo20114

        Uusia Mitokondriotauteja tulee lisää.
        Andokrinologiset muutokset:
        Lyhytkasvuisuus
        Infertiiliys
        Viivästynyt puberteetti
        Hypoparatyreoosi
        Diabetes

        Diabetes lisääntyy hurjasti eikä luonnonvalinta sitä poista. Vähänkään populaatioteoriaa tuntevana ymmärrät taudin viimein saavuttavan kaikki jälkeläiset. Periytyvyys menee läpi koko populaation.

        Laji ei todellakaan voi hyvin vaan rappeutuu. On tulossa yhä riippuvaisemmaksi lääkkeistä.


        Taudit lisääntyvät eikä mitokondrio saa mistään uuutaa

        Diabeteksen puhkeamiseen vaikuttaa paljon elintaso ja elintavat.


      • Anonyymi
        agnoskepo kirjoitti:

        "Laji ei todellakaan voi hyvin vaan rappeutuu."

        Niinkö artikkelissa luki? Minä ymmärsin artikkelin aivan toisin.

        " Niihin sairastutaan niin myöhään, etteivät ne ole kovin merkityksellisiä luonnonvalinnan kannalta."
        Eikä myöskään populaation kelpoisuuden kannalta. Luonnonvalinta ei poista virheitä, jotka vaikuttavatn vasta lisääntymisiän jälkeen, mutta eipä niillä ole myöskään sanottavaa rappeuttavaa vaikutusta. Ei ainakan niin, että populaation säilyvyys olisi uhattuna.

        " On tulossa yhä riippuvaisemmaksi lääkkeistä."
        Emme olisi sen riippuvaisempia lääkkeistä kuin ennenkään, jos odotettavissa oleva elinikämme heiluisi 40 vuoden paikkeilla, eikä 80 vuoden. Lääkkeillä on kyetty nostamaan elinikää, mikä samalla altistaa vanhenemisen tuomille lisääntyville sairauksille. Populaation genomin rappeutumisen kanssa tälläkään ei ole mitään tekemistä.

        "Edelleen, pystytkö osoittamaan, että yhdessäkään sukupuuttotapauksessa olisi primäärisyynä ollut rappeutuminen?"

        Tuohon kysymykseen et kommentoinut mitenkään, vaikka olit lainannut sitä juttusi alkuun.

        " On tulossa yhä riippuvaisemmaksi lääkkeistä."
        Emme olisi sen riippuvaisempia lääkkeistä kuin ennenkään, jos odotettavissa oleva elinikämme heiluisi 40 vuoden paikkeilla, eikä 80 vuoden. Lääkkeillä on kyetty nostamaan elinikää, mikä samalla altistaa vanhenemisen tuomille lisääntyville sairauksille. Populaation genomin rappeutumisen kanssa tälläkään ei ole mitään tekemistä.

        Allergiat, 1 tyyopin diapetes ja geneetisiä rsairauksia tulee kaikenaikaa lisää. Lääketiede ei kohta puysy perässä.


        Mammutin jälkeläiset eivät saavutaneet lisääntymisikää ruuansulatus,karvapeiteen yms geneetisensulamisen takia. Jokaisella nisäkkäällä on genominen elinkaari. Tuon nopeuden määrää juurikin tuo mutatoituminen. Informaatio vähenee, surkastumat. Tai genomi tulee toimintakyvyttömäksi.
        https://www.indiatoday.in/fyi/story/woolly-mammoths-suffered-genetic-meltdown-before-extinction-963734-2017-03-03


      • Anonyymi
        Anonyymi kirjoitti:

        " On tulossa yhä riippuvaisemmaksi lääkkeistä."
        Emme olisi sen riippuvaisempia lääkkeistä kuin ennenkään, jos odotettavissa oleva elinikämme heiluisi 40 vuoden paikkeilla, eikä 80 vuoden. Lääkkeillä on kyetty nostamaan elinikää, mikä samalla altistaa vanhenemisen tuomille lisääntyville sairauksille. Populaation genomin rappeutumisen kanssa tälläkään ei ole mitään tekemistä.

        Allergiat, 1 tyyopin diapetes ja geneetisiä rsairauksia tulee kaikenaikaa lisää. Lääketiede ei kohta puysy perässä.


        Mammutin jälkeläiset eivät saavutaneet lisääntymisikää ruuansulatus,karvapeiteen yms geneetisensulamisen takia. Jokaisella nisäkkäällä on genominen elinkaari. Tuon nopeuden määrää juurikin tuo mutatoituminen. Informaatio vähenee, surkastumat. Tai genomi tulee toimintakyvyttömäksi.
        https://www.indiatoday.in/fyi/story/woolly-mammoths-suffered-genetic-meltdown-before-extinction-963734-2017-03-03

        Tässäkään tapauksessa mammutit eivät kuolleet sukupuuttoon varsinaisesti geneettisen rappeutumisen takia. Ne kuolivat koska niille sopivat elinalueet kutistuivat mikä aiheutti nälkäkuolemia, ihminenkin vaikutti asiaan karsimalla kantaa jne. Kanta kävi liian pieneksi jotta se olisi pystynyt pitämään itsensä yllä. Mammutin kaltaisella hitaasti lisääntyvällä lajilla jääkauden jälkeinen ilmastonmuutos ja sitä seurannut kasvillisuusvyöhykkeiden valtava muutos oli erityisen kohtalokas.


      • Anonyymi
        Anonyymi kirjoitti:

        " On tulossa yhä riippuvaisemmaksi lääkkeistä."
        Emme olisi sen riippuvaisempia lääkkeistä kuin ennenkään, jos odotettavissa oleva elinikämme heiluisi 40 vuoden paikkeilla, eikä 80 vuoden. Lääkkeillä on kyetty nostamaan elinikää, mikä samalla altistaa vanhenemisen tuomille lisääntyville sairauksille. Populaation genomin rappeutumisen kanssa tälläkään ei ole mitään tekemistä.

        Allergiat, 1 tyyopin diapetes ja geneetisiä rsairauksia tulee kaikenaikaa lisää. Lääketiede ei kohta puysy perässä.


        Mammutin jälkeläiset eivät saavutaneet lisääntymisikää ruuansulatus,karvapeiteen yms geneetisensulamisen takia. Jokaisella nisäkkäällä on genominen elinkaari. Tuon nopeuden määrää juurikin tuo mutatoituminen. Informaatio vähenee, surkastumat. Tai genomi tulee toimintakyvyttömäksi.
        https://www.indiatoday.in/fyi/story/woolly-mammoths-suffered-genetic-meltdown-before-extinction-963734-2017-03-03

        Esimerkiksi allergioiden lisääntymisen on osoitettu johtuvan ylihygienisestä ympäristöstä lapsuusaikana.
        Väitit ykköstyypin diabeteksen lisääntyvän. Millä perusteella?
        Diabetesliitto:
        "Vuosittain tai edes muutaman vuoden välein päivittyvää tilastoa diabeetikoiden määrästä ei Suomessa ole. "
        Entä näyttö geneettisten sairauksien lisääntymisestä? Mistä se tilasto löytyy?


    • Milloin aiot panna toimeksi eli kumota evoluutioteorian tieteellisin argumentein?

      • Anonyymi

        Kun ei se pseudotiedemies osaa eikä kykene.


      • Anonyymi
        Anonyymi kirjoitti:

        Kun ei se pseudotiedemies osaa eikä kykene.

        Miten kumota teoria jota ei ole todistettu? Oletko evoluutioteoreetikko jossain yliopistossa ja haluat, huu haa teoriasi kumoamista.


      • Anonyymi
        Anonyymi kirjoitti:

        Miten kumota teoria jota ei ole todistettu? Oletko evoluutioteoreetikko jossain yliopistossa ja haluat, huu haa teoriasi kumoamista.

        Evoluutioteorian paikkansapitävyydestä on tiedemaailmassa konsensus ja sitä tukee laaja kirjo poikkitieteellisiä havaintoja. Joudut siis hylkäämään monet muutkin tieteen osa-alueet kuin vain biologian, kun lähdet taistelemaan evoluutioteoriaa vastaan. Mutta kyllä mielelläni näkisin miten sinä evoluutioteorian kumoat. Ja voin auttaa sinua alkuun tehtävässäsi. Alla lista, jotka osoittamalla saat falsifoitua evoluutioteorian. Kun olet tehtävän suorittanut, niin annatko vielä havainnollistavat todisteet sinun yliluonnollisesta entiteetistäsi. Saa suorittaa.
        - Geenifrekvenssit eivät populaatiossa muutu ajan kuluessa
        - Geenifrekvenssien muutoksella ei ole vaikutusta populaation selviytymiselle
        - luonnonvalintaa ei ole olemassa
        - DNA:ta ei ole olemassa
        - DNA ei koodaa eliöiden fenotyyppiä
        - mutaatioita ei ole olemassa
        - DNA ei voi muuttua
        - jälkeläiset eivät muistuta keskimääräisesti enempää vanhempiaan kuin muita populaation yksilöitä
        - universumi tai maa on 6000 vuotta
        - abiogeneesi on mahdotonta
        - alkumaan olosuhteet olivat hapettavia
        - fossiilikerrostumissa on kaikki lajeja sekaisin kaikilta aikakausilta (esim. jänisfossiili pre-kambrikauden kerrostumissa)
        - todellisia kimeerisiä eliöitä on olemassa: kentaurit, merenneidot jne.
        - havaintoja, että eliöitä luodaan tyhjästä nykypäivänä
        - on olemassa vaihtoehtoinen selitys bakteerien antibioottiresistenssille
        - Ihmisten kromosomi 2. ja suurten ihmisapinoiden kahden kromosomien yhtäläisyydet johtuvat jostain muusta kuin ihmisen kromosomin fuusioitumisesta, miten tarkalleen ottaen sentromeeri- ja telomeeriosat ovat päätyneet kromosomin keskelle?


      • Anonyymi
        Anonyymi kirjoitti:

        Evoluutioteorian paikkansapitävyydestä on tiedemaailmassa konsensus ja sitä tukee laaja kirjo poikkitieteellisiä havaintoja. Joudut siis hylkäämään monet muutkin tieteen osa-alueet kuin vain biologian, kun lähdet taistelemaan evoluutioteoriaa vastaan. Mutta kyllä mielelläni näkisin miten sinä evoluutioteorian kumoat. Ja voin auttaa sinua alkuun tehtävässäsi. Alla lista, jotka osoittamalla saat falsifoitua evoluutioteorian. Kun olet tehtävän suorittanut, niin annatko vielä havainnollistavat todisteet sinun yliluonnollisesta entiteetistäsi. Saa suorittaa.
        - Geenifrekvenssit eivät populaatiossa muutu ajan kuluessa
        - Geenifrekvenssien muutoksella ei ole vaikutusta populaation selviytymiselle
        - luonnonvalintaa ei ole olemassa
        - DNA:ta ei ole olemassa
        - DNA ei koodaa eliöiden fenotyyppiä
        - mutaatioita ei ole olemassa
        - DNA ei voi muuttua
        - jälkeläiset eivät muistuta keskimääräisesti enempää vanhempiaan kuin muita populaation yksilöitä
        - universumi tai maa on 6000 vuotta
        - abiogeneesi on mahdotonta
        - alkumaan olosuhteet olivat hapettavia
        - fossiilikerrostumissa on kaikki lajeja sekaisin kaikilta aikakausilta (esim. jänisfossiili pre-kambrikauden kerrostumissa)
        - todellisia kimeerisiä eliöitä on olemassa: kentaurit, merenneidot jne.
        - havaintoja, että eliöitä luodaan tyhjästä nykypäivänä
        - on olemassa vaihtoehtoinen selitys bakteerien antibioottiresistenssille
        - Ihmisten kromosomi 2. ja suurten ihmisapinoiden kahden kromosomien yhtäläisyydet johtuvat jostain muusta kuin ihmisen kromosomin fuusioitumisesta, miten tarkalleen ottaen sentromeeri- ja telomeeriosat ovat päätyneet kromosomin keskelle?

        Konsensus on vain evouskovaisten kesken

        Luonnonvalinta ei tuota mitään uutta. Luonnonvalinta tuhoaa myös hyvät muutokset joita on vähemmän kuin huonoja.

        DNA muutuu mutaatioiden johdosta ja rapautaa vähitellen koko genomin.

        Abiogeneeesi on todellakin mahdoton. Eliömaailma rappeutuu. John C. Sanford. Eliömaailma ei kehity eikä ole syntynyt tyhjästä.


      • Anonyymi
        Anonyymi kirjoitti:

        Konsensus on vain evouskovaisten kesken

        Luonnonvalinta ei tuota mitään uutta. Luonnonvalinta tuhoaa myös hyvät muutokset joita on vähemmän kuin huonoja.

        DNA muutuu mutaatioiden johdosta ja rapautaa vähitellen koko genomin.

        Abiogeneeesi on todellakin mahdoton. Eliömaailma rappeutuu. John C. Sanford. Eliömaailma ei kehity eikä ole syntynyt tyhjästä.

        Edelleen, evoluutio luonnossa ei ole mielipideasia. Jos sinulla ei ole tarjota tutkimuksia väitteittesi tueksi, niin olet aika heikoilla jäillä. John C. Sanford on kreationisti eikä hänkään ole evoluutioteoriaa kumonnut, saatikka antanut minkäänlaista havainnollistavaa näyttöä siitä, että kaiken taustalla olisi jokin yliluonnollinen entiteetti. Joten yrittäisitkö uudestaan.


      • Anonyymi
        Anonyymi kirjoitti:

        Edelleen, evoluutio luonnossa ei ole mielipideasia. Jos sinulla ei ole tarjota tutkimuksia väitteittesi tueksi, niin olet aika heikoilla jäillä. John C. Sanford on kreationisti eikä hänkään ole evoluutioteoriaa kumonnut, saatikka antanut minkäänlaista havainnollistavaa näyttöä siitä, että kaiken taustalla olisi jokin yliluonnollinen entiteetti. Joten yrittäisitkö uudestaan.

        John C. Sanfordi kirja kumoaa liiteineen evoluution. biologisella, kemiallisella ja matemaatisella tasolla. Elutiota ei ole.

        cernissä todistetiin higgsin bosonin olemassa olo joka kumoaa alkuräjähdysteorian. Jäljelle jää Albert Einstein suhteelisuusteorian avaruuden kaateutuminen, joka estää alkuräjähdyksen. Tilalle on kehittetty jo uusia teorioita.


      • Anonyymi
        Anonyymi kirjoitti:

        Evoluutioteorian paikkansapitävyydestä on tiedemaailmassa konsensus ja sitä tukee laaja kirjo poikkitieteellisiä havaintoja. Joudut siis hylkäämään monet muutkin tieteen osa-alueet kuin vain biologian, kun lähdet taistelemaan evoluutioteoriaa vastaan. Mutta kyllä mielelläni näkisin miten sinä evoluutioteorian kumoat. Ja voin auttaa sinua alkuun tehtävässäsi. Alla lista, jotka osoittamalla saat falsifoitua evoluutioteorian. Kun olet tehtävän suorittanut, niin annatko vielä havainnollistavat todisteet sinun yliluonnollisesta entiteetistäsi. Saa suorittaa.
        - Geenifrekvenssit eivät populaatiossa muutu ajan kuluessa
        - Geenifrekvenssien muutoksella ei ole vaikutusta populaation selviytymiselle
        - luonnonvalintaa ei ole olemassa
        - DNA:ta ei ole olemassa
        - DNA ei koodaa eliöiden fenotyyppiä
        - mutaatioita ei ole olemassa
        - DNA ei voi muuttua
        - jälkeläiset eivät muistuta keskimääräisesti enempää vanhempiaan kuin muita populaation yksilöitä
        - universumi tai maa on 6000 vuotta
        - abiogeneesi on mahdotonta
        - alkumaan olosuhteet olivat hapettavia
        - fossiilikerrostumissa on kaikki lajeja sekaisin kaikilta aikakausilta (esim. jänisfossiili pre-kambrikauden kerrostumissa)
        - todellisia kimeerisiä eliöitä on olemassa: kentaurit, merenneidot jne.
        - havaintoja, että eliöitä luodaan tyhjästä nykypäivänä
        - on olemassa vaihtoehtoinen selitys bakteerien antibioottiresistenssille
        - Ihmisten kromosomi 2. ja suurten ihmisapinoiden kahden kromosomien yhtäläisyydet johtuvat jostain muusta kuin ihmisen kromosomin fuusioitumisesta, miten tarkalleen ottaen sentromeeri- ja telomeeriosat ovat päätyneet kromosomin keskelle?

        " Ihmisten kromosomi 2. ja suurten ihmisapinoiden kahden kromosomien yhtäläisyydet johtuvat jostain muusta kuin ihmisen kromosomin fuusioitumisesta, miten tarkalleen ottaen sentromeeri- ja telomeeriosat ovat päätyneet kromosomin keskelle? "


        Kromosomien oletetaan fuusioituneen ”head-to-head” eli päistään.11 Muilla lajeilla tunnetaan kromosomifuusioita, mutta normaalisoluista ei ainuttakaan head-to-head-fuusiota. Yleensä fuusio tapahtuu lähellä ainakin toisen kromosomin sentromeeria, jos on tapahtuakseen, ja fuusiokohdan pitäisi sisältää ns. satellitti-DNA:ta, jota ei ihmisen kromosomin 2 oletetusta fuusiokohdasta löydy.

        https://www.youtube.com/watch?v=zi8FfMBYCkk

        https://answersingenesis.org/genetics/dna-similarities/chromosome-tales-and-importance-biblical-worldview/

        Väite on patavanhoilleinen ja darvinistinen. Kukaan järkevä ei enää väitä fuusioitumista tapahtuneen.
        Sananlasku yksi hullu kysyy enenmän kuin kymmenen viisata pystyy
        vastaaamaan, pitää paikkansa.


      • Anonyymi kirjoitti:

        Miten kumota teoria jota ei ole todistettu? Oletko evoluutioteoreetikko jossain yliopistossa ja haluat, huu haa teoriasi kumoamista.

        no miten tieteellisen teorian voi kumota uskonnollisella saastalla?


      • Anonyymi
        Anonyymi kirjoitti:

        John C. Sanfordi kirja kumoaa liiteineen evoluution. biologisella, kemiallisella ja matemaatisella tasolla. Elutiota ei ole.

        cernissä todistetiin higgsin bosonin olemassa olo joka kumoaa alkuräjähdysteorian. Jäljelle jää Albert Einstein suhteelisuusteorian avaruuden kaateutuminen, joka estää alkuräjähdyksen. Tilalle on kehittetty jo uusia teorioita.

        Sanford ei kumonnut evoluutioteoriaa. Väite on hölmö.

        Miten Higgsin bosonin löytyminen kumoaa alkuräjähdysteorian? En ole kuullut ydenkään fyysikon väittävän tuollaista. Higgsin bosonin löytymisestä on jo viisi vuotta ja BB-teoria voi paksusti.

        Avaruuden kaartuminen ei estä alkuräjähdystä. Itse asiassa inflaatio noudattaa melko hyvin De Sitter eksponentiaalisesti laajenevaa avaruutta, joka on yleisen suhtiksen ratkaisu.

        BB:n korvaavia ja sitä täydentäviä teorioita on aina pyritty kehittämään. Mitään BB:n korvaavaa ei ole löydetty.


      • Anonyymi
        ravenlored kirjoitti:

        no miten tieteellisen teorian voi kumota uskonnollisella saastalla?

        Ei mitenkään.
        Sentromeeri- ja telomeeriosat päätyvät kromosomin keskelle, kun fuusioituminen tapahtuu. Muuta selitystä on turha hakea, mutta pitäähän kressujen yrittää.

        Linkkisi vei uskonnolliselle sivustolle, jolla ei ole nimestään huolimatta mitään tekemistä tieteen kanssa.
        "In a biblical worldview, it is possible for a chromosome to have resulted from the fusion of two smaller chromosomes. "
        " If someone presents evidence that appears to contradict the biblical history, one excellent response is to investigate more fully to see what is going on."

        Kuulostaako muka tieteelliseltä?


      • Anonyymi
        Anonyymi kirjoitti:

        " Ihmisten kromosomi 2. ja suurten ihmisapinoiden kahden kromosomien yhtäläisyydet johtuvat jostain muusta kuin ihmisen kromosomin fuusioitumisesta, miten tarkalleen ottaen sentromeeri- ja telomeeriosat ovat päätyneet kromosomin keskelle? "


        Kromosomien oletetaan fuusioituneen ”head-to-head” eli päistään.11 Muilla lajeilla tunnetaan kromosomifuusioita, mutta normaalisoluista ei ainuttakaan head-to-head-fuusiota. Yleensä fuusio tapahtuu lähellä ainakin toisen kromosomin sentromeeria, jos on tapahtuakseen, ja fuusiokohdan pitäisi sisältää ns. satellitti-DNA:ta, jota ei ihmisen kromosomin 2 oletetusta fuusiokohdasta löydy.

        https://www.youtube.com/watch?v=zi8FfMBYCkk

        https://answersingenesis.org/genetics/dna-similarities/chromosome-tales-and-importance-biblical-worldview/

        Väite on patavanhoilleinen ja darvinistinen. Kukaan järkevä ei enää väitä fuusioitumista tapahtuneen.
        Sananlasku yksi hullu kysyy enenmän kuin kymmenen viisata pystyy
        vastaaamaan, pitää paikkansa.

        ".Kukaan järkevä ei enää väitä fuusioitumista tapahtuneen."

        Ei kukaan muu kuin molekyylibiologit.
        Kukaan järkevä ihminen ei usko kreationistien höpinöitä.


      • Anonyymi kirjoitti:

        Miten kumota teoria jota ei ole todistettu? Oletko evoluutioteoreetikko jossain yliopistossa ja haluat, huu haa teoriasi kumoamista.

        "Oletko evoluutioteoreetikko jossain yliopistossa ja haluat, huu haa teoriasi kumoamista."

        Jos olisin evoluutiobiologi, haluaisin taatusti kaataa evoluutioteorian, koska sillä saisi mainetta ja kunniaa loppuiäkseen. Evoluutioteoriaa ei kuitenkaan kaada kukaan ROTin kaltainen höpisijä eikä se ole "huu haa teoria" vaan niin vankasti todennettu kuin tieteellinen teoria ylipäänsä voi olla.


      • Anonyymi
        Anonyymi kirjoitti:

        Sanford ei kumonnut evoluutioteoriaa. Väite on hölmö.

        Miten Higgsin bosonin löytyminen kumoaa alkuräjähdysteorian? En ole kuullut ydenkään fyysikon väittävän tuollaista. Higgsin bosonin löytymisestä on jo viisi vuotta ja BB-teoria voi paksusti.

        Avaruuden kaartuminen ei estä alkuräjähdystä. Itse asiassa inflaatio noudattaa melko hyvin De Sitter eksponentiaalisesti laajenevaa avaruutta, joka on yleisen suhtiksen ratkaisu.

        BB:n korvaavia ja sitä täydentäviä teorioita on aina pyritty kehittämään. Mitään BB:n korvaavaa ei ole löydetty.

        Vertaisarvioitu julkaisu väittää jotain muuta.

        https://www.space.com/13094-accelerating-universe-dark-energy-illusion.html

        Pimeää energiaa eikä ainetta löydy. Cern on vetänyt vesiperän.


      • Anonyymi
        tarkistuskysymys kirjoitti:

        "Oletko evoluutioteoreetikko jossain yliopistossa ja haluat, huu haa teoriasi kumoamista."

        Jos olisin evoluutiobiologi, haluaisin taatusti kaataa evoluutioteorian, koska sillä saisi mainetta ja kunniaa loppuiäkseen. Evoluutioteoriaa ei kuitenkaan kaada kukaan ROTin kaltainen höpisijä eikä se ole "huu haa teoria" vaan niin vankasti todennettu kuin tieteellinen teoria ylipäänsä voi olla.

        DNA ja solu kuin muna ja kana

        Elämän alkuperään liittyy ratkeamaton "muna vai kana -ongelma": Perimämolekyylejä ei synny elävien solujen ulkopuolella; oli siis oltava olemassa soluja ennen tällaisten molekyylien syntyä. Toisaalta kuitenkaan solukaan ei voi edes pysyä hengissä saati kopioitua ilman sen rakenneinformaatiota kantavia perimämolekyylejä: perimämolekyylejä oli siis oltava ennen ensimmäisen solun syntyä.

        Miten evolutionismi sitten yrittää ratkaista tämän ongelman? "Maton alle lakaisemisen" (ongelman "unohtamisen" tai vähättelyn) lisäksi keinona on lähinnä sellaisen väitteen esittäminen, että ennen nykyisenlaista elämää on täytynyt olla toisenlaista elämää, joka jotenkin on mahdollistanut nykyisenlaisen elämän synnyn. Tämä on kuitenkin täysin hypoteettista spekulointia, sillä tällaisesta "esielämästä" ei ole löytynyt jälkeäkään eikä liioin osata sanoa, millaista se olisi voinut olla. On kyllä esitetty "RNA-maailman" tapaisia hypoteeseja, mutta ne eivät toimi periaatteessa eivätkä käytännössä. Umpikujaan johtaneita ehdotuksia siis riittää, mutta varsinainen vastaus tähän ongelmaan on jälleen kerran "tieteen pelisääntöjä noudattavan tulevan tutkimuksen tuloksiin" vetoaminen – viime kädessä tyydytään siis toistelemaan väitettä, että sokea naturalistinen usko on perustava tieteellinen hyve, jonka noudattaminen vielä kerran palkitaan; parempaakaan argumenttia ei näet kerta kaikkiaan ole käytettävissä.44

        DNA, informaatio ja proteiinisynteesikoneisto kuin tietokonemuisti, ohjelma ja kääntäjä

        Geneettisen koodin alkuperä on evolutionismille ratkeamaton arvoitus: Jotta perimämolekyylit voisivat kantaa solulle käyttökelpoista informaatiota, niiden lukemiseen on oltava valmiit solunsisäiset koneistot. Nämä kuitenkin rakennetaan elävissä soluissa jo olevien koneistojen ja niiden perimämolekyyleissä jo olevien koneistonrakenneohjeiden avulla.

        Sekä perimämolekyylit että niiden tulkintakoneisto että niiden sisältämä (mm. tulkintakoneiston) rakenneinformaatio vaativat näin ollen kukin kahden muun olemassaolon voidakseen itse täyttää tehtävänsä ja ajan mittaan kopioitua:
        1.Aineellisia prosesseja ohjaavaa informaatiota ei soluissa voi olla ilman perimämolekyylejä ja tulkintakoneistoa; sehän vaatii talletustilan ja lukulaitteiston.
        2.Tulkintakoneistoa taas ei voi olla ilman näitä molekyylejä ja niihin koodattua informaatiota; sehän vaatii toimiakseen, mistä lukea ja mitä sieltä lukea – ja kopioituakseen, että voi sieltä lukea omat kokoonpano-ohjeensa!
        3.Perimämolekyylejä ei puolestaan voi olla ilman informaatiota ja tulkintakoneistoa; niiden rakentaminenhan vaatii nämä – ja taaskin informaatiolle on sitä paitsi lisäehtoja: sen avulla pitää rakentaa nimenomaan sellainen molekyylikoneisto, joka sisältää virheenkorjausmekanismein varustetun DNA-molekyylien luotettavan kopiointimekanismin.

        Kun kaikki tämä on käytettävissä, biosfäärissä kaikki toimii, elää, kopioituu, kasvaa ja lisääntyy. Jos tätä kaikkea ei olisi, ei olisi mitään biosfääriäkään – ei edes "alustavaa biosfääriä", ei mitään sen tapaistakaan. "Ei kahta kolmannetta" siis pitää tässä kyllä paikkansa, mutta vain siinä mielessä, että kaikki kolme tarvitaan, ei siinä mielessä, että mikään niistä voisi kahden muun jatkoksi jälkikäteen jotenkin vain "ilmaantua".

        Geneettisen koodin alkuperä ei selity "luonnollisesti"

        Geneettinen koodikieli ei periaatteessa kehity vaan pysyy kaiken aikaa samana. Evolutionismin mukaan se on pysynyt sellaisenaan biologisen elämän historian alusta asti: samaa koodia tottelevat niin bakteerit kuin valaatkin. Miten tällainen koodi sitten olisi voinut alun perin syntyä suunnittelemattomasti ja tarkoituksettomasti? Evoluutioselityshän ei periaatteessakaan voi toimia, ellei ole jatkuvaa pientä muuntelua antamassa luonnonvalinnalle ylipäänsä joitain vaihtoehtoja valittaviksi. Ainakaan evoluutio ei siis voi mitenkään selittää geneettisen koodin syntyä, koodin kun piti jo olla olemassa ennen kuin oletettu evoluutio pääsi edes alkamaan.

        Miten evolutionistit sitten tulevat toimeen tämän ongelman kanssa? Tyypilliseen tapaansa he ovat tässäkin kohden "siivilöineet hyttysen mutta nielleet kamelin" olettamalla geneettisen koodin alun perin vain jotenkin itsestään ilmaantuneen ja käyttämällä sen (oletettua) eliökunnanlaajuista universaalisuutta sitten polveutumisopin perusteluna. Ideana siis on keskittyä siihen näkökohtaan, että geneettisen koodin universaalisuus sopii hyvin yhteen polveutumisopin kanssa, ja unohtaa sen syntyyn liittyvä kolossaalinen selittämättömyys. "Jotenkinhan sen täytyi tapahtua, emmekä vielä kaikkea tiedä, mutta evoluutio on joka tapauksessa ainoa tieteellinen ajattelutapa" jne.


      • Anonyymi kirjoitti:

        Vertaisarvioitu julkaisu väittää jotain muuta.

        https://www.space.com/13094-accelerating-universe-dark-energy-illusion.html

        Pimeää energiaa eikä ainetta löydy. Cern on vetänyt vesiperän.

        "Pimeää energiaa eikä ainetta löydy. Cern on vetänyt vesiperän. "

        Cernin LHC rakennettiin alunperin Higgsin bosonin etsimiseen, kvarkki-guluonipalsman tutkimiseen ja lottopottina olisi pidetty standardimallin jälkeisen fysiikan löytymistä.
        Kaksi ensimmäistä toteutui varsin hyvin. Kolmannesta ei merkkejäkään ole löydetty.
        Pimeä aine olisi varmaan ollut jonkinlainen ekstra, mutta sitä varten LHC:a ei todellakaan rakennettu. Pimeän energian selvittäminen LHC:a olisi ollut teoreettisestikin aivan mahdotonta.
        LHC ei todellakaan vetänyt vesiperää, vaan on tuottanut juuri sen tiedon, mitä siltä odotettiin. Datassa on analysoitavaa useille vuosille, jos ei vuosikymmenille tästä eteenpäinkin.

        Luin linkittämäsi artikelin. Se on jo 8 vuotta vanha spekulaatio, eikä ole tietääkseni saanut vahvistusta. Ei se, että artikkeli on vertaisarvioitu, takaa johtopäätösten oikeellisuutta. Se takaa vain sen, ettei artikkeli ole aivan sutta ja segundaa. Cernin LHC kiihdytintä ei kyseisessä artikkelissa edes mainittu.

        Taidat olla tieteellisestä keskustelusta melkoisen pihalla.


      • Anonyymi kirjoitti:

        DNA ja solu kuin muna ja kana

        Elämän alkuperään liittyy ratkeamaton "muna vai kana -ongelma": Perimämolekyylejä ei synny elävien solujen ulkopuolella; oli siis oltava olemassa soluja ennen tällaisten molekyylien syntyä. Toisaalta kuitenkaan solukaan ei voi edes pysyä hengissä saati kopioitua ilman sen rakenneinformaatiota kantavia perimämolekyylejä: perimämolekyylejä oli siis oltava ennen ensimmäisen solun syntyä.

        Miten evolutionismi sitten yrittää ratkaista tämän ongelman? "Maton alle lakaisemisen" (ongelman "unohtamisen" tai vähättelyn) lisäksi keinona on lähinnä sellaisen väitteen esittäminen, että ennen nykyisenlaista elämää on täytynyt olla toisenlaista elämää, joka jotenkin on mahdollistanut nykyisenlaisen elämän synnyn. Tämä on kuitenkin täysin hypoteettista spekulointia, sillä tällaisesta "esielämästä" ei ole löytynyt jälkeäkään eikä liioin osata sanoa, millaista se olisi voinut olla. On kyllä esitetty "RNA-maailman" tapaisia hypoteeseja, mutta ne eivät toimi periaatteessa eivätkä käytännössä. Umpikujaan johtaneita ehdotuksia siis riittää, mutta varsinainen vastaus tähän ongelmaan on jälleen kerran "tieteen pelisääntöjä noudattavan tulevan tutkimuksen tuloksiin" vetoaminen – viime kädessä tyydytään siis toistelemaan väitettä, että sokea naturalistinen usko on perustava tieteellinen hyve, jonka noudattaminen vielä kerran palkitaan; parempaakaan argumenttia ei näet kerta kaikkiaan ole käytettävissä.44

        DNA, informaatio ja proteiinisynteesikoneisto kuin tietokonemuisti, ohjelma ja kääntäjä

        Geneettisen koodin alkuperä on evolutionismille ratkeamaton arvoitus: Jotta perimämolekyylit voisivat kantaa solulle käyttökelpoista informaatiota, niiden lukemiseen on oltava valmiit solunsisäiset koneistot. Nämä kuitenkin rakennetaan elävissä soluissa jo olevien koneistojen ja niiden perimämolekyyleissä jo olevien koneistonrakenneohjeiden avulla.

        Sekä perimämolekyylit että niiden tulkintakoneisto että niiden sisältämä (mm. tulkintakoneiston) rakenneinformaatio vaativat näin ollen kukin kahden muun olemassaolon voidakseen itse täyttää tehtävänsä ja ajan mittaan kopioitua:
        1.Aineellisia prosesseja ohjaavaa informaatiota ei soluissa voi olla ilman perimämolekyylejä ja tulkintakoneistoa; sehän vaatii talletustilan ja lukulaitteiston.
        2.Tulkintakoneistoa taas ei voi olla ilman näitä molekyylejä ja niihin koodattua informaatiota; sehän vaatii toimiakseen, mistä lukea ja mitä sieltä lukea – ja kopioituakseen, että voi sieltä lukea omat kokoonpano-ohjeensa!
        3.Perimämolekyylejä ei puolestaan voi olla ilman informaatiota ja tulkintakoneistoa; niiden rakentaminenhan vaatii nämä – ja taaskin informaatiolle on sitä paitsi lisäehtoja: sen avulla pitää rakentaa nimenomaan sellainen molekyylikoneisto, joka sisältää virheenkorjausmekanismein varustetun DNA-molekyylien luotettavan kopiointimekanismin.

        Kun kaikki tämä on käytettävissä, biosfäärissä kaikki toimii, elää, kopioituu, kasvaa ja lisääntyy. Jos tätä kaikkea ei olisi, ei olisi mitään biosfääriäkään – ei edes "alustavaa biosfääriä", ei mitään sen tapaistakaan. "Ei kahta kolmannetta" siis pitää tässä kyllä paikkansa, mutta vain siinä mielessä, että kaikki kolme tarvitaan, ei siinä mielessä, että mikään niistä voisi kahden muun jatkoksi jälkikäteen jotenkin vain "ilmaantua".

        Geneettisen koodin alkuperä ei selity "luonnollisesti"

        Geneettinen koodikieli ei periaatteessa kehity vaan pysyy kaiken aikaa samana. Evolutionismin mukaan se on pysynyt sellaisenaan biologisen elämän historian alusta asti: samaa koodia tottelevat niin bakteerit kuin valaatkin. Miten tällainen koodi sitten olisi voinut alun perin syntyä suunnittelemattomasti ja tarkoituksettomasti? Evoluutioselityshän ei periaatteessakaan voi toimia, ellei ole jatkuvaa pientä muuntelua antamassa luonnonvalinnalle ylipäänsä joitain vaihtoehtoja valittaviksi. Ainakaan evoluutio ei siis voi mitenkään selittää geneettisen koodin syntyä, koodin kun piti jo olla olemassa ennen kuin oletettu evoluutio pääsi edes alkamaan.

        Miten evolutionistit sitten tulevat toimeen tämän ongelman kanssa? Tyypilliseen tapaansa he ovat tässäkin kohden "siivilöineet hyttysen mutta nielleet kamelin" olettamalla geneettisen koodin alun perin vain jotenkin itsestään ilmaantuneen ja käyttämällä sen (oletettua) eliökunnanlaajuista universaalisuutta sitten polveutumisopin perusteluna. Ideana siis on keskittyä siihen näkökohtaan, että geneettisen koodin universaalisuus sopii hyvin yhteen polveutumisopin kanssa, ja unohtaa sen syntyyn liittyvä kolossaalinen selittämättömyys. "Jotenkinhan sen täytyi tapahtua, emmekä vielä kaikkea tiedä, mutta evoluutio on joka tapauksessa ainoa tieteellinen ajattelutapa" jne.

        Montako kertaa teille pitää kertoa, ettei alkuperäinen replikaattori ollut lähellekään niin monimutkainen kuin solu?


      • Anonyymi
        agnoskepo kirjoitti:

        Montako kertaa teille pitää kertoa, ettei alkuperäinen replikaattori ollut lähellekään niin monimutkainen kuin solu?

        Lisäksi tämä kyseinen anonyymi näköjään kopioi oikeastaan kaikki tekstinsä joko Apowikistä, creation.com-, answersingenesis.com- tai intelligentdesign.fi-sivustoilta. Kun kaikki ammennettava tieto näköjään suodatetaan uskonnollisen paatoksen kautta niin on hyvin vaikea ymmärtää miten tiedettä oikeasti tehdään ja mitä tuloksia tieteen avulla on saavutettu. Ja sitten pystytään kirkkain silmin vielä ymmärtämään väärin, tai oikeastaan valehtelemaan siitä mitä tutkijat ovat tutkimusten osalta raportoineet. ROT on tästä lähes täydellinen esimerkki. Tuskin tulemme näkemään sitä päivää, että kreationistit ymmärtäisivät sen, että ensimmäinen kopioitumiseen kykenevät organismi ollut nykyisin tuntemamme solun kaltainen eliö. Mutta kaipa se heidän kohdallaan toimii paremmin niin, että epärehellisyys maan perii.


    • "Jokainen syntyvä lapsi tuo 100-200 geenivirhettä perimään ja koska roska-DNA:ta ei ole, johtavat nämä geenivirheet väistämättömään ja nopeaan geneettiseen rappeutumiseen."

      Ei johda. Informaatiotieteessä tunnetaan lukemattomia virheenkorjausmenetelmiä.

      "Ja koska evoluutiota ei siis täten tapahdu"

      Kyllä tapahtuu. Jokainen syntyvä lapsi todistaa evoluution olevan edelleen käynnissä oleva prosessi.

      "Raamattu on totta ensimmäisestä viimeiseen jakeeseen."

      Taasko se maailma on litteä ja jumala on muunsukupuolinen ja apinan näköinen sadisti?

      • Anonyymi

        kirjoitin tuossa aiemmin.

        Mitokondriotaudit

        https://www.duodecimlehti.fi/lehti/1992/6/duo20114

        Mitokondriotauteihin luetaan myös ATP-syntaasi häiriöt. Soluu ei saa energiaa. Ihminen tarvitsee oman painonsa verran ATP:ä joka vuorokausi pitääkseen toimintojaan yllä.

        Meillähän ei ole mitään huolta. virheenkorjausmekanismit korjaavat periytyvät taudit vai kuinka.? Ennen noita mekanismeja kehitys oli hurjaa.?Tapahtui eliöräjähdys, kun mutaatioita ei korjannut mikään moottoriproteiini?


      • Anonyymi

        Virheenkorjausmekanismeihinkin tulee vikoja.


      • Anonyymi
        Anonyymi kirjoitti:

        Virheenkorjausmekanismeihinkin tulee vikoja.

        "Virheenkorjausmekanismeihinkin tulee vikoja."

        Näin on. Mistäpä se syöpä muutoin syntyisi. solun itseohjautuva kuolema on häiriintynyt. Myös muut kasvua säätelevät tekijät. Solu ajetaan alas ja osat käytetään uudelleen. Eliöllä on kehittynyt jätehuolto järjestelmä joka pilkkoo aminohapot alkutekijöihinsä ja kierrättää ne uusiokäyttöön.


      • Anonyymi

        "Ei johda. Informaatiotieteessä tunnetaan lukemattomia virheenkorjausmenetelmiä."

        Voitko kertoa mitä informaatio tieteelä tarkoitat ja missä kohtaa. Luonnossa ei ole mitään mekanismia tuotaa mitään informaatiota. Seti projekti ei ole tuonut muuta kuin taustakohinaa. Tuosta kohinasta ei syny mitään.


      • Anonyymi kirjoitti:

        "Ei johda. Informaatiotieteessä tunnetaan lukemattomia virheenkorjausmenetelmiä."

        Voitko kertoa mitä informaatio tieteelä tarkoitat ja missä kohtaa. Luonnossa ei ole mitään mekanismia tuotaa mitään informaatiota. Seti projekti ei ole tuonut muuta kuin taustakohinaa. Tuosta kohinasta ei syny mitään.

        "Voitko kertoa mitä informaatio tieteelä tarkoitat"

        https://en.wikipedia.org/wiki/Information_science

        "Luonnossa ei ole mitään mekanismia tuotaa mitään informaatiota. Seti projekti ei ole tuonut muuta kuin taustakohinaa. Tuosta kohinasta ei syny mitään."

        Mahdollisimman korkea informaatiosisältö on pelkkää kohinaa: https://en.wikipedia.org/wiki/Shannon–Hartley_theorem


      • Anonyymi
        M-Kar kirjoitti:

        "Voitko kertoa mitä informaatio tieteelä tarkoitat"

        https://en.wikipedia.org/wiki/Information_science

        "Luonnossa ei ole mitään mekanismia tuotaa mitään informaatiota. Seti projekti ei ole tuonut muuta kuin taustakohinaa. Tuosta kohinasta ei syny mitään."

        Mahdollisimman korkea informaatiosisältö on pelkkää kohinaa: https://en.wikipedia.org/wiki/Shannon–Hartley_theorem

        Voit yhtä hyvin laitaa auton sulatusuuniin ja valaa sen lattialle ja väitää siinä olevan auton, lentokoneen tai mootoripyörän.

        Tuosta kasasta ei sytoiminnallista konetta
        https://creation.com/laws-of-information-1
        https://creation.com/laws-of-information-2

        Väiteesi on absurdi.


      • Anonyymi kirjoitti:

        Voit yhtä hyvin laitaa auton sulatusuuniin ja valaa sen lattialle ja väitää siinä olevan auton, lentokoneen tai mootoripyörän.

        Tuosta kasasta ei sytoiminnallista konetta
        https://creation.com/laws-of-information-1
        https://creation.com/laws-of-information-2

        Väiteesi on absurdi.

        "Väiteesi on absurdi."

        Älä linkitä niitä roska-artikkeleja, ne eivät liity informaatiotieteeseen mikä tutkii informaatiota.

        Kohina on sitä informaatiota ja tämä on helposti todistettavissa.

        Voit kokeilla kotitietokoneella tehdä jollain "muistio" ohjelmalla tiedoston missä copypastella vedetty pelkkää "A" kirjainta.

        Sitten tallentaa mikrofonilla kohinaa samankokoiseksi tiedostoksi.

        Seuraavaksi pakkaa molemmat tiedostot tietokoneesta löytyvällä "zip" ohjelmalla mikä tiivistää tiedostot niin nähdään, että kohinaa pakkautuu paljon heikommin kuin pelkkää A-kirjainta järjestyksessä oleva tiedosto. Kohinassa siis on enemmän informaatiota.

        "Voit yhtä hyvin laitaa auton sulatusuuniin ja valaa sen lattialle ja väitää siinä olevan auton, lentokoneen tai mootoripyörän."

        Jos valaa metallin harkoiksi, siinä on enemmän järjestystä ja vähemmän informaatiota. Jos höyrystän sen metallin ydinräjähteellä, siinä on enemmän epäjärjestystystä ja enemmän informaatiota.

        "Tuosta kasasta ei sytoiminnallista konetta"

        Saa jos materian järjestää toiminnalliseksi. Materian järjestäytyminen tarvitsee energiaa kuten termodynamiikan laki kertoo ja vähentää entropiaa, ja se vähentää informaatiota koska lisääntynyt entropia on suurempi informaatiomäärä.


      • Anonyymi
        M-Kar kirjoitti:

        "Väiteesi on absurdi."

        Älä linkitä niitä roska-artikkeleja, ne eivät liity informaatiotieteeseen mikä tutkii informaatiota.

        Kohina on sitä informaatiota ja tämä on helposti todistettavissa.

        Voit kokeilla kotitietokoneella tehdä jollain "muistio" ohjelmalla tiedoston missä copypastella vedetty pelkkää "A" kirjainta.

        Sitten tallentaa mikrofonilla kohinaa samankokoiseksi tiedostoksi.

        Seuraavaksi pakkaa molemmat tiedostot tietokoneesta löytyvällä "zip" ohjelmalla mikä tiivistää tiedostot niin nähdään, että kohinaa pakkautuu paljon heikommin kuin pelkkää A-kirjainta järjestyksessä oleva tiedosto. Kohinassa siis on enemmän informaatiota.

        "Voit yhtä hyvin laitaa auton sulatusuuniin ja valaa sen lattialle ja väitää siinä olevan auton, lentokoneen tai mootoripyörän."

        Jos valaa metallin harkoiksi, siinä on enemmän järjestystä ja vähemmän informaatiota. Jos höyrystän sen metallin ydinräjähteellä, siinä on enemmän epäjärjestystystä ja enemmän informaatiota.

        "Tuosta kasasta ei sytoiminnallista konetta"

        Saa jos materian järjestää toiminnalliseksi. Materian järjestäytyminen tarvitsee energiaa kuten termodynamiikan laki kertoo ja vähentää entropiaa, ja se vähentää informaatiota koska lisääntynyt entropia on suurempi informaatiomäärä.

        Selvennetään vielä kerran. Ota nurmikonsiemenseosta ja laita sen veden kanssa tehosekoittimeen ja sekoita. Sekoittamisen jälkeen siemenet ovat epäjärjestyksessä. Hajonneita eikä tuosta enää kasva ruohoa. Hajottamalla alkuaineiksi tuosta sopasta ei synny enää mitään. Noista ei tule koskaan männyn tai koivun siementä, koska tieto ja välineet rakentamiseen puutuu.

        Esimerkkisi tarvitsee vastaan ottajan, jona toimii tietokone. Tarvitset kaikkea linkittämässäni artikkelissa mainitsemia komponentteja pelkästään tietokoneessasi. Tarvitset kokeeseesi järjestäytynyttä teknologiaa jota ei luonnossa synny. Tuo informaatio kaaos ei tuota mitään. Siihen ei synny ensimmäistäkään kirjainta. Voit katsoa vanhaa putki TV.n kuvaa Nykyaikana sinne ei tule edes ohjelmaa, vaan vain taustakohina Ei sine synny sitä ensimmäistäkään merkkiä. Teet tietokonetta apuna käyttäen tuon merkin. Kaaos ei sisällä mitään järjestäytynyttä informaatiota. Ajatuksesi on filosofinen musta-aukko. Siitä ei pääse mikään ulos.

        Sekoitat täydellisesti informaatiotasot.
        Tilastollinen taso, kielioppitaso, merkitystaso, vaikutustaso, tarkoitustaso

        Kohina ei ole informaatiota. Kohina on kaaosta.

        Voit olla varma että tulen linkittämän vielä monesti nuo linkit. nämä viestinnän,kommunikoinnin ja informaation kaava on universaali. Se toimii kaikkialla missä osapuolet kommunikoivat.


      • Anonyymi kirjoitti:

        Selvennetään vielä kerran. Ota nurmikonsiemenseosta ja laita sen veden kanssa tehosekoittimeen ja sekoita. Sekoittamisen jälkeen siemenet ovat epäjärjestyksessä. Hajonneita eikä tuosta enää kasva ruohoa. Hajottamalla alkuaineiksi tuosta sopasta ei synny enää mitään. Noista ei tule koskaan männyn tai koivun siementä, koska tieto ja välineet rakentamiseen puutuu.

        Esimerkkisi tarvitsee vastaan ottajan, jona toimii tietokone. Tarvitset kaikkea linkittämässäni artikkelissa mainitsemia komponentteja pelkästään tietokoneessasi. Tarvitset kokeeseesi järjestäytynyttä teknologiaa jota ei luonnossa synny. Tuo informaatio kaaos ei tuota mitään. Siihen ei synny ensimmäistäkään kirjainta. Voit katsoa vanhaa putki TV.n kuvaa Nykyaikana sinne ei tule edes ohjelmaa, vaan vain taustakohina Ei sine synny sitä ensimmäistäkään merkkiä. Teet tietokonetta apuna käyttäen tuon merkin. Kaaos ei sisällä mitään järjestäytynyttä informaatiota. Ajatuksesi on filosofinen musta-aukko. Siitä ei pääse mikään ulos.

        Sekoitat täydellisesti informaatiotasot.
        Tilastollinen taso, kielioppitaso, merkitystaso, vaikutustaso, tarkoitustaso

        Kohina ei ole informaatiota. Kohina on kaaosta.

        Voit olla varma että tulen linkittämän vielä monesti nuo linkit. nämä viestinnän,kommunikoinnin ja informaation kaava on universaali. Se toimii kaikkialla missä osapuolet kommunikoivat.

        "Selvennetään vielä kerran. Ota nurmikonsiemenseosta ja laita sen veden kanssa tehosekoittimeen ja sekoita. Sekoittamisen jälkeen siemenet ovat epäjärjestyksessä. Hajonneita eikä tuosta enää kasva ruohoa."

        Totta, ja informaation määrä kasvaa tuossa. Entropiassa informaation määrä kasvaa.

        Näiden korjaaminen vaatii materian järjestäytymistä mikä vaatii energiaa ja siinä yhteydessä informaation määrä vähenee.

        "Hajottamalla alkuaineiksi tuosta sopasta ei synny enää mitään. Noista ei tule koskaan männyn tai koivun siementä, koska tieto ja välineet rakentamiseen puutuu."

        Hajottamalla alkuaineiksi yhdeksi puuroksi, informaation määrä kasvaa ja vaatii enemmän energiaa järjestää. Ja kyllä muuten alkuaineiksi hajotetuista järjestäytyy kokoaika uudestaan elämään. Katsoo vaikka pellolle miten siellä vilja kasvaa niin siinä nimenomaan järjestäytyy sekaisin mullassa olevat hiiliyhdisteet. Järjestäytyminen tapahtuu auringon energialla.

        "Sekoitat täydellisesti informaatiotasot.
        Tilastollinen taso, kielioppitaso, merkitystaso, vaikutustaso, tarkoitustaso"

        Sekoitat informaatioon jotain semantiikkaa. Informaatiolla ei tarvitse olla mitään merkitystä, sillä ei tarvitse olla tarkoitusta, sen ei tarvitse vaikuttaa mihinkään, sen ei tarvitse kommunikoida mihinkään eikä sen tarvitse tuottaa mitään. Informaation määrää mitataan bitteinä.

        https://en.wikipedia.org/wiki/Information

        "Information can be thought of as the resolution of uncertainty"

        Eli suomennettuna, informaatiota voi ajatella epävarmuuden ratkaisuna. Mitään merkityksiä se ei tarvitse.


    • Anonyymi

      Ei sillä ole merkitystä, jos todistelet evoluutioteoriaa vastaan ja luulet sen kumoavadi. Evoluutioteorian tutkiminen ja kehitys jatkuu todisteluistasi huolimatta.

      Ja se on hyvä. Ihmiskunnan on hyvä ymmärtää mistä on tultu ja mitä voidaan arvioida tulevan tapahtumaan.

      Uskonnot ovat pyrkineet estämään tiedettä joko väkisin tai omilla teorioilla. Näin myös on evoluutioteorian osalta, tarjoamalla luomista.

      Valitettavasti uskonnot ovat vuosisadoista toiseen joutuneet tunnustamaan olevansa väärässä. Tosin ei kaikki yksilötasolla ymmärrä tätä. Aurinko ei kierrä maapalloa :)

      Mikään ei estä evoluutioteorian tutkimusta ja kehittymistä. Mm. DNA tutkimus tuottaa hämmästyttäviä tuloksia eri tasoilla koko ajan.

      • Anonyymi

        Umpilisäkkeen piti olla todiste evolutiosta. Toisin on. Umpilisäkeitä on poistettu muiden operaatioiden yhteydessä ja hyvin heoisin perustein. Nyt sitten tehdään suolistobakteerien siirtoja.

        Evo uskovainen muutaa kantaansa vuosikymnmenestä toiseen.

        Mitä paskaa vielä tulemme näkemäänkään, evo uskovaisten alkaessa manipuloida luontoa, rakennellen sinne omia virityksiään.


      • Anonyymi
        Anonyymi kirjoitti:

        Umpilisäkkeen piti olla todiste evolutiosta. Toisin on. Umpilisäkeitä on poistettu muiden operaatioiden yhteydessä ja hyvin heoisin perustein. Nyt sitten tehdään suolistobakteerien siirtoja.

        Evo uskovainen muutaa kantaansa vuosikymnmenestä toiseen.

        Mitä paskaa vielä tulemme näkemäänkään, evo uskovaisten alkaessa manipuloida luontoa, rakennellen sinne omia virityksiään.

        Suolistobakteerien siirtoa tehdään vaikeissa tapauksissa, on umpilisäkettä poistettu tai ei. Joskus poistoja on tehty heppoisin perustein, mutta ei tarvitse mennä kauas ajassa taaksepäin, kun umpisuolen tulehdukseen vielä kuoltiin.


      • Anonyymi
        Anonyymi kirjoitti:

        Suolistobakteerien siirtoa tehdään vaikeissa tapauksissa, on umpilisäkettä poistettu tai ei. Joskus poistoja on tehty heppoisin perustein, mutta ei tarvitse mennä kauas ajassa taaksepäin, kun umpisuolen tulehdukseen vielä kuoltiin.

        Onko sillä umpisuolella funtio vai ei? Esimerkkejä löytyy evouskovaisten toilailuista vaikka kuinka. Ei ole kauan kun sinisilmäinen arjalainen oli se ihmisrodun kirkkain helmi. Stalin opiskeli papiksi ja hylkäsi uskonnon. Pahuus sai vallan ilman estoja.

        Nyky aika tuo tullessaan estottoman väkivallan ja pahuuden. ihminen vastaa vain itseleen.


      • Anonyymi kirjoitti:

        Onko sillä umpisuolella funtio vai ei? Esimerkkejä löytyy evouskovaisten toilailuista vaikka kuinka. Ei ole kauan kun sinisilmäinen arjalainen oli se ihmisrodun kirkkain helmi. Stalin opiskeli papiksi ja hylkäsi uskonnon. Pahuus sai vallan ilman estoja.

        Nyky aika tuo tullessaan estottoman väkivallan ja pahuuden. ihminen vastaa vain itseleen.

        SE, onko umpilisäkkeellä funktio vai ei, ei ratkaise, onko se evolutiivinen surkastuma. Funktio voi muuttua evoluution myötä. Nykyinen näkemys taitaa olla se, että evoluutio on kehittänyt umpisuolen ja umpilisäkkeen.

        Wiki:
        "Heather F. Smith ja William Parkerin tutkimusryhmä totesi, että tutkituista 361 nisäkäslajista noin 50:ltä löytyi umpisuoli ja umpilisäke, ja se on kehittynyt evoluution kuluessa itsenäisesti 32 kertaa."


      • Anonyymi
        agnoskepo kirjoitti:

        SE, onko umpilisäkkeellä funktio vai ei, ei ratkaise, onko se evolutiivinen surkastuma. Funktio voi muuttua evoluution myötä. Nykyinen näkemys taitaa olla se, että evoluutio on kehittänyt umpisuolen ja umpilisäkkeen.

        Wiki:
        "Heather F. Smith ja William Parkerin tutkimusryhmä totesi, että tutkituista 361 nisäkäslajista noin 50:ltä löytyi umpisuoli ja umpilisäke, ja se on kehittynyt evoluution kuluessa itsenäisesti 32 kertaa."

        Surkastuma on tiedon häviämistä. Tieto ei lisäänny itsenäisesti.


      • Anonyymi
        Anonyymi kirjoitti:

        Surkastuma on tiedon häviämistä. Tieto ei lisäänny itsenäisesti.

        Mitä tarkoitat väitteellä, että tieto ei lisäänny itsenäisesti?


    • Anonyymi

      "Evoluutioteorian voi kumota nykyään helposti vain yhdellä lauseella"

      Ei kyllä voi. Teorioita kumotaan kehittämällä parempi teoria. Noin yleisesti ottaen kokonaisia teorioita ei kuitenkaan ole kovin paljon kumottu, lähinnä vain rakennettu teorioiden laajennuksia, jotka pätevät laajemmin.

      • Anonyymi

        Tuollaiset väitteet evoluutioteorian kumoamisesta osoittavat vain, kuin kaukana henkilö on tervejärkisestä ajattelusta. Ei taida puolillakaan inkkareista mela ulottua veteen asti.


    Ketjusta on poistettu 4 sääntöjenvastaista viestiä.

    Luetuimmat keskustelut

    1. KUPSinpelaaja vangittu törkeästä rikoksesta

      Tänään tuli uutinen että Kupsin sopimuspelaajs vangittu törkeästä rikoksesta epäiltynä. Kuka pelaaja kysressä ja mikä ri
      Kuopio
      19
      1661
    2. Taasko se show alkaa

      Koo osottaa taas mieltään
      Ikävä
      29
      1319
    3. Minun oma kaivattuni

      Ei ole mikään ilkeä kiusaajatyyppi, vaan sivistynyt ja fiksu sekä ystävällinen ihminen, ja arvostan häntä suuresti. Raka
      Ikävä
      64
      1224
    4. Miksi ihmeessä nainen seurustelit kanssani joskus

      Olin ruma silloin ja nykyisin vielä rumempi En voi kuin miettiä että miksi Olitko vain rikki edellisestä suhteesta ja ha
      Ikävä
      11
      1132
    5. Persut nimittivät kummeli-hahmon valtiosihteeriksi!

      Persujen riveistä löytyi taas uusi törkyturpa valtiosihteeriksi! Jutun perusteella järjenjuoksu on kuin sketsihahmolla.
      Perussuomalaiset
      28
      1066
    6. Tervehdys!

      Sä voit poistaa nää kaikki, mut mä kysyn silti A:lta sen kokemuksia sun käytöksestä eron jälkeen. Btw, miks haluut sabot
      Turku
      65
      1046
    7. Onko ministeri Juuso epäkelpo ministerin tehtäviensä hoitamiseen?

      Eikö hänellä ole kompetenttia hoitaa sosiaali- ja terveysministetin toimialalle kuuluvia ministerin tehtäviä?
      Perussuomalaiset
      10
      1045
    8. Elia tulee vielä

      Johannes Kastaja oli Elia, mutta Jeesus sanoi, että Elia tulee vielä. Malakian kirjan profetia Eliasta toteutuu kokonaan
      Helluntailaisuus
      30
      1009
    9. Paske perse pillu

      Kulli seksi hiv
      Ikävä
      6
      1003
    10. Sakarjan kirjan 6. luku

      Jolla korva on, se kuulkoon. Sain profetian 22.4.2023. Sen sisältö oli seuraava: Suomeen tulee nälänhätä niin, että se
      Profetiat
      6
      1001
    Aihe